Главная Юзердоски Каталог Трекер NSFW Настройки

Математика

Ответить в тред Ответить в тред
Check this out!
<<
Назад | Вниз | Каталог | Обновить | Автообновление | 501 61 130
МАТЕМАТИКА ДЛЯ НАЧИНАЮЩИХ N+1 Аноним 28/11/17 Втр 22:39:58 29047 1
tumblroujih3a5i[...].jpg 52Кб, 500x500
500x500
В этом треде мы изучаем математику. Если ты школьник или студент, и у тебя есть трудности с задачей, то здесь тебе помогут её решить или хотя бы скажут, в каком направлении двигаться для её решения. Чем более чётко и конкретно ты опишешь суть своих затруднений, тем выше твой шанс на содержательный ответ.

Основные списки литературы:
http://pastebin.com/raw/4iMjfWAf - classic
http://pastebin.com/raw/4FngRj6n - dxdy

Архив тредов (там же остальные списки литературы и полезные ссылки):
https://pastebin.com/raw/qhs0WNbY
Аноним 05/05/24 Вск 01:42:48 114737 2
Аноним 05/05/24 Вск 14:03:26 114749 3
>>114721
Так там дают теорию множеств вначале. То что ты знал в школе ты тоже когда-то не знал. Новое всегда учить тяжело, но со временем оно становится элементарным.
Бтв конкретно про множества, один важный факт, которые многие не упоминают: элементы в множестве обязательно различимы. В множестве не может содержаться копии. $A=\{x,x\}=\{x\}$
>Хочу вкатиться в матан
Можешь взять Ленга: Short Calculus: The Original Edition of “A First Course in Calculus”, там нет множеств.
Аноним 05/05/24 Вск 14:30:18 114750 4
>>114712
Молодец что развивается. Лучше так, чем офигевать от обилия ёбл мразотных ТП, которых тьма.
Аноним 05/05/24 Вск 15:18:56 114751 5
>>114731
нечитабельная душная дичь для профессиональных аутистов

мимо гуманитарий
Аноним 05/05/24 Вск 16:02:54 114752 6
IMG202405051558[...].jpg 815Кб, 2560x620
2560x620
IMG202405051346[...].jpg 2460Кб, 2592x4608
2592x4608
Подскажите пожалуйста, как решается №3а и желательно №3б (пикрил). Пробовал сделать через обобщенную сферическую замену, но получилась лажа
Аноним 05/05/24 Вск 21:12:19 114756 7
>>114751
ты задавал вопрос:
>Двач, что читать, чтоб легче понимать зорича?
? спасибо за отзыв, но очется конкретизации. и еще интересно было бы увидеть именно отзыв анона, обратившегося за советом.
вообще-то, том 1 ЭЭМ отлично подходит как звено между математикой средней и высшей школы. непонятна претензия:
>нечитабельная душная дичь для профессиональных аутистов
, потому что изложение как раз-таки минималистичное по объему самого предмета, стройное и последовательное, при этом педагогически мастерское.
если же претензия относится к самому математическому предмету - то чего же от математики еще ожидать? математика ведь требует вездесущей концентрации и аккуратности. любая вольность нарушает саму ее методологию.
>>114749
>Бтв конкретно про множества, один важный факт, которые многие не упоминают: элементы в множестве обязательно различимы. В множестве не может содержаться копии. [math]A = { x, x } = { x }[/math]
зач ты так написал? как раз лучше бы наоборот написал: могут модержаться копии
Аноним 05/05/24 Вск 21:15:25 114757 8
>>114756
сори фикс
>[math]A = \{ x, x \} = \{ x \}[/math]
Аноним 05/05/24 Вск 22:31:58 114758 9
>>114736
>>114737
Завтра напишу, у меня сегодня неполадки с компьютером.
Аноним 06/05/24 Пнд 10:26:18 114762 10
>>114722
а если предположить что пи нормально число, получается функция везде = 1?
Аноним 06/05/24 Пнд 11:13:22 114763 11
>>114762
Нет. если $\pi = 3.14\dots(9)$ то да, а если нет, то нет.
Тащемта $\pi$ обычное число. Из $\mathbb{R}$ и без всяких прикольчиков.
Аноним 06/05/24 Пнд 11:26:40 114764 12
image.png 19Кб, 1694x94
1694x94
Аноним 06/05/24 Пнд 14:00:17 114765 13
Screenshot2024-[...].jpg 495Кб, 1080x1130
1080x1130
Аноним 06/05/24 Пнд 18:04:01 114766 14
Аноним 06/05/24 Пнд 21:46:00 114767 15
image.png 135Кб, 1182x617
1182x617
image.png 29Кб, 533x340
533x340
Так, столкнулся с проблемкой, есть одна статья, и авторы делят точки наблюдения в двумерном пространстве на квадранты (4, отмечены пунктирными линиями), и считают в каждом квадранте Integrated probability

Что такое это самое Integrated probability в русской терминологии?

Потому что в другой работе на аналогичную тему (пик 2) авторы не используют этот термин, и, судя по всему, просто посчитали число точек разных классов в квадранте.

В первом же пике 1 оранжевая точка это 0.03 в квадранте, 2 - уже 0.12, 3 - 0.23 ....
Аноним 06/05/24 Пнд 21:49:09 114768 16
>>114767
Ты предлагаешь гадать по картинкам? Статью в студию
Аноним 06/05/24 Пнд 21:59:20 114769 17
>>114768
1) Fasano–Franceschini Test: an Implementation of a 2-Dimensional Kolmogorov–Smirnov test in R by Elan Ness-Cohn, Rosemary Braun

2) fasano.franceschini.test: An Implementation of a Multidimensional KS Test in R

Оригинальная работа

3) A multidimensional version of the Kolmogorov-Smirnov test
Fasano, G. ; Franceschini, A.

Если лень искать работы, дропнул в файл https:// filetransfer.io/data-package/oTKJjRLf#link
pa
Аноним 06/05/24 Пнд 22:50:34 114770 18
>>114769
Так, полистал я твои статьи, спасибо что удобно всё собрал.
Точка, которая используется для разделения на квадранты, засчитывается в равной мере в каждый квадрант. Соответственно если оранжевых точек 10, и одна оранжевая точка используется как опорная, то мы её долю (1/10) делим между всеми квадрантами, получая 0.025 (или, округляя, 0.03).
Соответственно ноль точек это 0.025 (0.03 на рисунке), одна точка это 0.1+0.025=0.125, и так далее.

В статье #2 этого не сделано, что очень сомнительно, потому что в статье #3 написано, что нормируем на единицу. Но при больших n наверное похуй.
Аноним 07/05/24 Втр 01:55:34 114771 19
>>114764
Подловил. Я использовал нормальное в житейском смысле.
Аноним 07/05/24 Втр 08:21:30 114772 20
>>114770
Спасибо, я обратил внимание, что в первой статье опорная точка засчитывается в каждый квадрант по 0.03, но не додумался, что это просто четверть от 0.1. Потому что во второй работе они опорную точку не засчитывали никуда. Если что, обе статьи за авторством одних и тех же людей, вторая на год поновее.

В их понимании Integrated probability это просто доля точек в квадранте от общего числа? Потому что как по мне, так и стоило написать :)

Еще раз спасибо
Аноним 07/05/24 Втр 23:09:42 114774 21
Вот говорят в трехмерных комплексных числах нельзя было бы делить - а почему их не используют в теории колец, там же всё равно деления нет.
Аноним 08/05/24 Срд 01:52:25 114776 22
>>114774
Это не теория колец, а теория конечномерных алгебр, мы же хотим с поля $\mathbb{R}$ стартовать (если смотреть на алгебры над $\mathbb{Q}$, то там есть и трёхмерные алгебры с делением). Простейший пример трёхмерной алгебры над $\mathbb{R}$ это фактор $\mathbb{R}/f(x)$, где $\operatornam{deg} f(x)=3$ и старший коэффициент у него 1.
Насчёт полной классификации я не знаю, но трёхмерные ассоциативные (но не всегда коммутативные) над вещественными и комплексными числами вроде гуглятся легко.
Аноним 08/05/24 Срд 03:17:17 114779 23
>>114776
*$\mathbb{R}[x]/f(x)$, где $\operatorname{deg} f(x)=3$
Аноним 08/05/24 Срд 16:20:55 114781 24
image.png 10Кб, 497x312
497x312
а есть формула что бы найти все точки пересечения серых прямых красной?
Аноним 08/05/24 Срд 17:12:13 114782 25
>>114781
Серые прямые задаются уравнениями вида $y=m$ и $x=n$, если есть уравнение красной прямой, то можно решить системы уравнений, получатся ответы, зависящие от параметров m или n.
Аноним 08/05/24 Срд 17:13:13 114783 26
>>114782
Ну, если шаг по единичке. Если сетка другая, то просто чуть модифицировать нужно.
Аноним 08/05/24 Срд 20:18:40 114786 27
>>114782
А если красная не прямая а луч
Аноним 09/05/24 Чтв 02:50:08 114787 28
>>114786
Отбери потом подходящие решения неравенством.
Аноним 09/05/24 Чтв 09:27:13 114790 29
>>114787
а как выглядит уравнения отрезка? почему то не смог ничего найти
Аноним 09/05/24 Чтв 11:59:15 114791 30
>>114790
y=kx+b, x_1<x<x_2
Можно и параметрически через векторное уравнение задать.
Аноним 09/05/24 Чтв 12:10:51 114792 31
>>114791
не, я имел ввиду просто уравнение без долнительных неравенст
Аноним 09/05/24 Чтв 13:27:22 114795 32
>>114792
Простого уравнения тут нет, насколько мне известно.
Аноним 09/05/24 Чтв 15:09:30 114801 33
>>114792
отрезок не может представлять собой множество решений никакого приличного уравнения в силу теоремы о неявной функции

подобрать искусственный пример, конечно, можно: например, задать функцию кусочно неравенствами, а потом искать множество её нулей
Аноним 09/05/24 Чтв 16:08:15 114803 34
>>114801
>в силу
Чуть подробнее можно?
Аноним 09/05/24 Чтв 16:51:46 114804 35
Аноним 09/05/24 Чтв 20:30:19 114805 36
Двач, представим у нас есть не тригонометрических окружность, а квадрат. Из центра координат можем точно так же проводить отрезки до стороны квадрата. Как будет выглядеть функция, которая возвращает длину этого отрезка для заданного угла?
Аноним 09/05/24 Чтв 21:51:34 114806 37
>>114805
очевидно, она будет кусночно гладкая, а на кусках возрыащать тангенс или котангенс
Аноним 09/05/24 Чтв 22:01:14 114808 38
>>114806
тангенс или котангенс нам будет давать длину куска соответствующей стороны квадрата. А чтобы длинуьоткрзка до этой стороны получаить, надо теорему пифагора ещё, т.е корень из 1 - квадрат этого тангеса
Аноним 09/05/24 Чтв 22:06:01 114809 39
>>114806
А без кусочности никак?
(Центр квадрата совпадает с началом координат)
Аноним 09/05/24 Чтв 22:16:21 114810 40
>>114809
никак, квадрат он сам кусочный
Аноним 10/05/24 Птн 17:53:51 114818 41
cover.webp 149Кб, 594x781
594x781
>>29047 (OP)
Блядь у меня просто нет слов. У меня сгорела жопа. Ну почему блядь школьные учебники по матеше такие непонятные сука. Я уже неделю маюсь с функциями по алгебре 7кл. от макарычева.

Я не понимаю ни-фи-га что там написано. Пожалуйста накидайте ресурсов по математике для даунов.

Самое главное, что квадратные уравнения, дроби, модули я спокойно выучил благодаря сайту математика с нуля и там гораздо приятнее учиться. НО БЛЯДЬ эти учебники просто пиздец, по ним же невозможно что-то выучить.

мимо-заводчанин-33-годиков
Аноним 10/05/24 Птн 17:55:42 114819 42
Сука хоть английский учи нахуй. Почему у нас такой менталитет все нахуй усложнять, ебаный рот.
Аноним 10/05/24 Птн 18:14:44 114821 43
изображение.png 1116Кб, 1068x703
1068x703
>>114818
Напиши конкретно что тебе непонятно.
Так же, тебе не всегда должно быть всё понятно как только ты прочитал определение. Многие незнакомые вещи кажутся нереально сложными, но со временем думаешь как ты мог тупить, это ведь так элементарно. Но до многих "элементарных" человечество догадывалось сотни лет. Геометрия существовала буквально тысячи лет, но координаты изобрели не так уж и давно.
Аноним 10/05/24 Птн 18:32:13 114822 44
Безымянныdй.png 386Кб, 887x711
887x711
>>114821
Ну например задача 303. Я вообще в парагрфе не нашел как решать такие задачи.

Отдельно хочу отметь почему нахуй в примерах постоянно применяются кубы степени и модули, если по идее степени находятся в следующей главе для изучения. Будь я школьником то я бы вообще не понял и обосрался бы от того, что там находится какая-то ебанина над цифрой.
Аноним 10/05/24 Птн 18:45:42 114823 45
Ну вы только посмотрите как все легко написано тут про теорему виета
https://spacemath.xyz/teorema-vieta/
От простого к сложному. Стена текста объясняющее все и несколько однотипных задач для закрепления. А про удобство с моментальной возможностью проверки своего решения я уже не говорю.
Я бы и дальше учился по нему, но я уже прошел уроки. Я думал ладно пойду по учебникам. И о мое удивление, как же все там громоздко, официозно и непонятно.
Аноним 10/05/24 Птн 18:54:28 114824 46
Я вообще не понимаю почему у нас еще не создали единый портал обучения. Зачем тратиться на школы хуелы. Если можно создать единый сайт с простеньким дизайном без свистоперделок и кислотными цветами (хан академи по этой причине говно). Люди бы сами обучались в удобное им время. И если бы не понял тему, то не пришлось бы гоняться за преподом.

Только представьте как бы такой портал поднял бы уровень образованности населения.

Я нашел примерно такой сайт на ангельском https://www.ixl.com/math конечно он тоже цветастный но не настолько громоздкий как хан академи. Но он сука платный, а я гречневый.
Аноним 10/05/24 Птн 18:55:36 114825 47
>>114823
учебники бывают разные
что ты рвёшься как птеух-неосилятор
Аноним 10/05/24 Птн 18:59:22 114826 48
>>114825
Да я неосилирятор. Но учебники то везде такие мерзляк такой же, петерсон такой же, шень такой же. Сканави...
Единственный норм учебник который я читал был по геометрии от атанасяна. Реально там задания начинались буквально с начерти треугольник и только потом по чуть-чуть усложнялись. Мне даже геометрия больше нравится начала чем алгебра из-за легкости.
Аноним 10/05/24 Птн 19:04:33 114827 49
>>114826
ок, ты не нашёл приятного для тебя учебника.
ну и что с того? истерить теперь?
попробуй посмотреть видео на ютубе, может, там что-то ронравится
Аноним 10/05/24 Птн 19:05:43 114828 50
>>114827
Да, это был крик-отчаяния. От того как все устроено, ведь можно же по другому...
Аноним 10/05/24 Птн 19:09:53 114829 51
>>114828
ну ладно, покричи, раз уж так требуется..
Аноним 10/05/24 Птн 19:21:46 114831 52
>>114825
Мелкочмондель, ты еще не вскрылся? Это ты зря.
Аноним 10/05/24 Птн 19:31:30 114832 53
image.png 136Кб, 298x438
298x438
ALLO, MATH ETO TI?
PUCHK-PUCHK
ШИФ-ШИФ
ГРОООТ!!!


>>114818
ОТКРОЙ ДЛЯ СЕБЯ ПЕРВОКУЛЬТУРНУЮ МАТЕМАТИКУ, МОЖЕШЬ НАЧАТЬ С БУРБАКОВ. ПОТОМ МОЖНО К РАССЛОЕНИМИ И ПУЧКАМ ПЕРЕЙТИ. НА ВТОРУЮ КУЛЬТУРУ ТРАТИТЬ ВРЕМЯ НЕЛЬЗЯ.

>>114822
ТЫ ПРОСТО ПЕРВОКУЛЬТУРНЫЙ МАТЕМАТИК, ВТОРОКУЛЬТУРЩИКИ ЗАДАЧИ РЕШАЮТ, ЗАДАЧИ НЕ НУЖНЫ, УЧИТ ТЕОРИИ ПУУУЧЧККК! ЧИТАЙ МОДУЛИ НАД КОЛЬЦАМИ.

Такие дела, Привет.
Аноним 11/05/24 Суб 00:24:52 114837 54
>>114822
Начни с разбора того, что значит "точка принадлежит графику функции" и как вообще описываются точки, принадлежащие графику функции и что вообще такое график. Перечитай определения, скорее всего разберёшься.
Аноним 11/05/24 Суб 00:31:19 114838 55
>>114822
График это совокупность точек на координатной плоскости, координаты которых удовлетворяют данному уравнению (т.е. если вместо x подставить абсциссу точки, вместо y ординату, и равенство будет правильным, то точка принадлежит графику функции).
> по идее степени находятся в следующей главе для изучения
А там не написано "в прошлом классе вы уже познакомились со степенями, сейчас мы узнаем про них больше..."?
> мимо-заводчанин-33-годиков
Нахуя оно тебе надо?
Аноним 11/05/24 Суб 00:35:33 114839 56
1663764737890.png 1Кб, 100x100
100x100
Аноним 11/05/24 Суб 02:29:04 114840 57
>>114822
проходит ли график функции f через точку M(x, y)?
сначала проверь, находится ли абсцисса x в области определения Df. если x не находится в Df, то значит, график функции f не проходит через M.
если x находится в Df, то график функции f проходит через некую точку (x, fx). найдем fx сравним с ординатой y. если fx ≠ y, то значит, (x, fx) и M(x, y) - разные точки, и поскольку x = x' влечет за собой fx = fx', то график функции f не проходит через M. если же получим fx = y, то, очевидно, график функции f проходит через M(x, y) по определению графика.
Аноним 11/05/24 Суб 08:53:56 114844 58
2024-05-1112-42[...].png 707Кб, 1284x877
1284x877
2024-05-1112-51[...].png 119Кб, 569x711
569x711
>>114837
Да, ты прав. Нашел вот такой разбор задач по учебнику https://www.youtube.com/watch?v=iBFntORhe-w&list=PLtgE87VTmyY-B6Vc7Au9lZQHnouimupYq
>>114838
>А там не написано "в прошлом классе вы уже познакомились со степенями
Нет. Пикрил мерзляк матеша за 6 класс. Да и не знаю зачем ты споришь, если я написал, что степени проходят в следующей главе.
>> Нахуя оно тебе надо?
Ладно, чуть-чуть спиздел, в порядке анонимности. Я тот анон, выше, который хотел стать инженером. Мне 27. Матеша понравилась благодаря тому сайту, который легко все объясняет и я почему-то подумал, что так легко будет со всеми ресурсами/учебниками. Придется мешать учебники с ютубом.
>>114840
Блин вот вроде бы на интуитивном уровне понятно. Но мой бугурт в том, что школьные учебники не предназначены для самостоятельного освоения. По ним все понятно становится только с объяснениями от учителя.
>>114832
Вот бы до бурбаков дойти...

капча
Аноним 11/05/24 Суб 11:53:56 114853 59
>>114844
Может у тебя учебник хуёвый попался? Я когда матешу начинал учить читал чебники за 7-11 класс Алимова. И там всё понятно с теорией и задачами. Такой хуйни как у тебя не было. Попробуй их чекнуть.
Аноним 11/05/24 Суб 12:19:55 114854 60
>>114853
Да, чекал алимова, у него в 2 раза меньше страниц чем у макарычева. Ну даже не знаю, просто макарычева преподают у моего младшего брата вот и взял его. Один фиг я почему-то уверен, что все они одинаковые.
Аноним 11/05/24 Суб 13:26:48 114860 61
>>114822
Может это просто не твоё? Займись чем-нубдь другим. Программированием например.
>>114823
Просто ты ищешь книгу с рецептами, а не учебник.
>>114824
>Если можно создать единый сайт с простеньким дизайном без свистоперделок и кислотными цветами (хан академи по этой причине говно). Люди бы сами обучались в удобное им время.
Уже есть такое. Книги.
Аноним 11/05/24 Суб 13:59:32 114861 62
>>114860
А что плохого в книгах с рецептами? По-моему они намного лучше будут для меня на данном этапе.
Аноним 11/05/24 Суб 14:00:48 114862 63
>>114824
Ну повторять разжёванные только что шаблоны и обезьяна сможет.
То, что ты на задачке про точки графика функции пососал, показывает, насколько каловое это обучение.
Не, для ознакомления норм. Но голову рано или поздно тоже придётся включать.
> Люди бы сами обучались в удобное им время
Да, обязательно
Аноним 11/05/24 Суб 14:06:51 114863 64
>>114862
Я пососал на функциях потому, что функции для меня не разжёвывали. Дробно-рациональные уравнения из 8 класса я смог решить, потому что до этого мне разжевали уравнения на простых уравнениях до сложных.
Аноним 11/05/24 Суб 14:14:02 114864 65
>>114862
Может лучше хотя бы иметь минимальные навыки благодаря разжеванным материалам, чем иметь буквально ничего?
Аноним 11/05/24 Суб 16:15:31 114865 66
>>114864
найми репетитора и не страдай

можешь даже зумершу с мехмата
Аноним 11/05/24 Суб 16:49:13 114866 67
>>114864
Не ищи репетитора. Пользуйся гуглом, когда совсем ничего не понимаешь.
Хотя это временами полезно получать подсказки и указания, качественное улучшение понимания лучше всего происходит при самостоятельной работе, при разборе неизвестных концепций, при попытке выстрадать результат самому.
Аноним 11/05/24 Суб 17:13:40 114867 68
>>114866
Нет, над школьной матешой, репетитора я точно не буду нанимать. Так или иначе я пойму все, просто бомблю с того, что придется потратить больше усилий.
>>114865
Может когда придет время универского матеши. Хотя мне почему то кажется, что она мне легче будет даваться.
Аноним 11/05/24 Суб 18:35:14 114870 69
>>114867
>просто бомблю с того, что придется потратить больше усилий.
А что в этом удивительного?
усилия надо прилагать везде - будь то музыка, рисование, танцы или даже гей-порно
Аноним 12/05/24 Вск 01:34:52 114872 70
>>114866
в тред пиши и выделяй жирным вопрос:
>Изучаю программу за 7 класс. Прохожу функции. Помогите. Как узнать проходит ли график функции через точку?
ответим без проблем. вопрос только интереса и трудовременных ресурсов. компетентности хватит с лихвой.
если что - дублируй вопрос, переспрашивай. раз вопросы простые - разве можно оставлять без помощи?
Аноним 12/05/24 Вск 11:36:17 114874 71
>>114872
Окей, тогда вопрос такой: есть ли в современной теории гомотопий/inf-категорий модели, в которых место симплициальных множеств занимает какой-то другой функтор?
Аноним 12/05/24 Вск 12:25:06 114878 72
изображение.png 1075Кб, 1105x737
1105x737
изображение.png 48Кб, 463x65
463x65
>>114861
>А что плохого в книгах с рецептами? По-моему они намного лучше будут для меня на данном этапе.
Потому что ты не сможешь нигде то что ты выучил применить.
Ты можешь скажем выучить рецепт блюда и повторить его. Но полезнее подойти к этой ситуации с общей стороны, узнать, при какой температуре белок в мясе сворачивается и тд. Тогда ты сможешь нормально приготовить не только курицу в своей духовке, но и в чужой, и шашлыки и пр.
Ты просто не хочешь напрягаться и потому горишь на учебники. В твоем учебнике есть всё что нужно тчобы решить 303 задачу. Причём если ты решаешь упражнения подряд, то у тебя уже была такая же задача.
Аноним 12/05/24 Вск 15:38:09 114885 73
>>114874
На любой категории симплициальных объектов (в категории удовлетворяющей ряду слабых свойств, вроде (ко)полноты) можно задать модельную структуру Риди. Это должно быть в книге Квиллена по гомотопической алгебре.
Аноним 12/05/24 Вск 16:25:03 114887 74
>>29047 (OP)
Является ли операция это частным случаем функции?
Аноним 12/05/24 Вск 16:30:07 114888 75
Аноним 12/05/24 Вск 16:51:22 114890 76
>>114885
Ну я не про модельные структуры, я в более обывательском смысле говорю про модели. Ведь как в алгебраической топологии начали триангулировать, так до сих пор в самых продвинутых разделах на это идейно опираются по сути ведь. Вот мне и интересно, есть ли альтернатива.
Аноним 12/05/24 Вск 16:53:48 114891 77
>>114878
С блюдом хороший пример. Только он в обратную сторону работает. Чтобы научиться готовить, нужно множество раз приготовить по рецептам, уже со временем разбираясь, почему в рецепте делают именно вот так и что будет, если сделать иначе.
Аноним 12/05/24 Вск 17:30:03 114893 78
Нахуй вы дебилу этому отвечаете. Ему юзер-лгбт-френдли приложуха нужна, где как для детей всё поясняют и напрягаться не приходится.
Школьный учебник для него сложный бля. Не догадался как подставить цифры вместо букв.
Такие чмошники просто плакаться на двач прибегают, советы им не нужны
Аноним 12/05/24 Вск 17:43:26 114894 79
>>114890
>Ну я не про модельные структуры, я в более обывательском смысле говорю про модели.
Я не понимаю вопроса, сформулируй строже. Про "модели" $(\infty,1)$-категорий есть в книжке Riehl & Verity.
>Ведь как в алгебраической топологии начали триангулировать, так до сих пор в самых продвинутых разделах на это идейно опираются по сути ведь.
Мне кажется это всё исчерпывается предложением "модельная категория топ. пространств и модельная категория симп. множеств Квиллен-эквивалентны".
Аноним 12/05/24 Вск 18:23:11 114899 80
А нахуя определять сложение рекурсивно через ряд добавлений единицы, если по-факту формально то самое добавление единицы не определено? И можно ли вообще определить понятие числа без количества, я имею ввиду логически, а не формально
Аноним 12/05/24 Вск 18:27:32 114900 81
>>114899
>без количества
Без количества просто потому что оно по-хорошему требует в определении число, что порождает взаимную зависимость понятий, не имеющую смысла
Аноним 12/05/24 Вск 18:39:09 114901 82
>>114899
>логически, а не формально
Что это значит?
"Добавление единицы" определено - это отображение из множества натуральных чисел в множество натуральных чисел удовлетворяющее ряду свойств.
Аноним 12/05/24 Вск 18:46:33 114902 83
>>114901
>"Добавление единицы" определено - это отображение из множества натуральных чисел в множество натуральных чисел удовлетворяющее ряду свойств.

Ну разумеется я могу создать некую модель, в которой есть некое действие +: N -> N = n + 1, но как я докажу что (n+1) больше n без аксиом и без знания что такое число? По сути такое определения сложения просто прячет аксиоматику, и заместо n + m = n + m мы говорим, что n + m это n плюс m раз по 1.
Аноним 12/05/24 Вск 19:03:54 114903 84
>>114891
Да нифига. Я в ПапаДжонс работал. Сколько я домашних пицц не пробовал, все они сосали у пицц с доставок. На работе было всё так налажено, что красивую пиццу, как на картинке, и вкусную, мог сделать любой стажёр в первый же день. Ну, разве что бортик будет неровный, и ингридиенты нарезаны криво. Потому что всё было налажено в общем виде. Тесто настоялось нужное количество часов, печка разогрета до нужной температуры и конвеер настроен чтобы пицца грелась именно определенное количество времени.
Ты можешь задрочить 100500 рецептов. Всё чему ты научишься это быстро и красиво нарезать овощи и эффективно отделять мясо от кости, и то сомнительно без гайдов. И если попытаешься сделать что-то сам, то обосрешься, потому что у тебя не будет знаний, зачем нужно прогревать духовку, какая температура свертываемости белка, почему не стоит варить в бурлящей воде и пр и пр.
Любой нормальный учебник по кулинарии начинается с этих вещей, а не с рецептов. Благодаря этому пищевая промышленность тебе может скормить субпродукты, которыми 200-300+ лет назад брезговали даже нищие, даже заключенные, а ты добавки попросишь.

Затупы это нормально, даже на элементарных вещах. Но вместо рейджа и перечитывания параграфа до полного отупения нужно забить, отвлечься, походить, прилечь, попытаться самому обдумать. Если не получается, гуглишь решение, спрашиваешь на форумах.
Метод хуячить по рецептам кстати распространен у прогеров, у них с презрением относятся к изобретательству велосипедов, но нормальное обучение матеше и состоит из изобретения 10 велосипедов на дню. Тебе должно быть тяжело и непонятно. Если тебе не тяжело и не непонятно, то ты просто тратишь время.
Аноним 12/05/24 Вск 19:46:25 114904 85
16952046898002.jpg 56Кб, 590x493
590x493
Абсолютное большинство "учебников" это буквально автор навалял говна из жопы. Куда уже страдать еще сильнее?

https://youtu.be/BPT1z9MQP_w?t=176
Аноним 12/05/24 Вск 20:16:28 114906 86
>>114903
>Любой нормальный учебник по кулинарии
Реквестирую.
Аноним 12/05/24 Вск 20:43:06 114908 87
Screenshot 2024[...].png 22Кб, 555x309
555x309
Есть две задачи оптимизации. Как показать что w_1 и b_1 колинеарны соответственно w_2 и b_2? w_1 и x^{(i)} в этой задачи вектора одной размерности, остальное сколярные величины.
Для контекста: эти две задачи оптимизации решаются при исполнении алгоритма SVM. Уравнение w_1^T x^{(i)} + b_1 = 0 определяет гиперплоскость которая должна быть равна гиперплотности такого же уравнения с параметрами w_2, b_2 из второй задачи (поэтому соответствующие параметры из двух задач и колинеарны).
Аноним 12/05/24 Вск 22:19:53 114909 88
>>114874
Возможно то что тебе нужно это кубическая теория типов. Хотя сомневаюсь что ты о ней не слышал. Это какой то вопрос с подъебом / при чем тут какие то графики функций?
Аноним 12/05/24 Вск 22:32:07 114910 89
>>114909
> Это какой то вопрос с подъебом / при чем тут какие то графики функций?
Просто анон промахнулся с реплаем.
А я просто во время своего не особо длинного пока экскурса в высокий теоркат удивился, насколько все любят "треугольнички", как будто до сих пор это из самых первых шагов в комбинаторной топологии несут с собой.
Аноним 13/05/24 Пнд 05:27:35 114924 90
>>114874
я бы и рад вам ответить, и даже ради этого разобрался бы в вопросе. несмотря на обилие терминологии, звучит просто. да только какое отношение он имеет к нашему разговору?
>>114887
отображение, функция, операция - одно и то же
>>114899
>можно ли вообще определить понятие числа без количества, я имею ввиду логически, а не формально
прекрасный и интересный вопрос. я сам им задавался. и с радостью поделюсь своими размышлениями.
вероятно, дело не в неопределенности, а в различии между идеями конечного и бесконечного. а еще точнее - в свойствах обозримости и необозримости.
количество, выражаемое натуральным числом - это самый легкий пример обозримого объекта. (※ кажется, слово "обозримость" является переводом на русский язык некоего понятия некоего логика, вероятно Крайзеля. за соответствие не ручаюсь. просто для нас здесь это самое выразительное и подходящее слово.) обозримые объекты - это такие объекты, которые либо понятны сразу целиком, либо представимы как конечная последовательность понятных деталей. легкость именно натуральных чисел достигается за счет того, они возникают из единственного начального объекта (0 для [math]\N_0[/math] или 1 для [math]\N_1[/math]) и изменяются лишь в одном направлении - единственная унарная операция прибавления единицы.
прибавление единицы определено с помощью рекурсивной схемы конечной длины. но при этом прибавление единицы образует идею о бесконечном, которая не сводима к идее о конечном.
допустим, мы хотим составить некое общее понятие об обозримых объектах. как же их охарактеризовать, описать, представить?
если бы мы явно ограничили круг рассмотрения объектов до конечного, то мы могли бы считать, что в какой-то момент всевозможные объекты будут исчерпаны.
однако, не введя предположение о конечности бытия, мы не можем исключать, что из любого рассматриваемого объекта можно получить новый, добавив некую новую деталь. если к обозримому объекту добавить обозримую деталь, то получим новый обозримый объект. это получение новых объектов аозможно бесконечно. таким образом, множество всех обозримых (если угодно - конечных) объектов вообще говоря не будет конечным, и описание обозримости (как и конечности) возможно только в контексте абстрактной бесконечной вселенной.
так через понятие "любой" (достраивание любого объекта) мы приходим к понятию бесконечности. для формулирования бесконечности через конечную схему, в схеме мы используем понятие "любой" и индукцию существования новых объектов. в случае натуральных чисел: [math]"\text{для любого k} \quad k + 1 \ne 0"[/math], [math]для любых k, l из k + 1 = l + 1 следует k = l[/math], благодаря чему прибавление единицы всякий раз дает новое число, не встречавшееся прежде.
в схеме не используется "количество m", в ней используется возможность прибавления единицы к любому числу. что, в саою очередь, является не ограничением, а следствием не введения ограничения о конечности.
>>114910
вот оно что! что ж, на этот раз Вы меня подловили, мусье
Аноним 13/05/24 Пнд 09:11:44 114926 91
nsin(x) - pcos(x) = a
как такие штуки решать?
Аноним 13/05/24 Пнд 11:03:25 114927 92
>>114926
через формулу Эйлера, как и любую тригонометрию
Аноним 13/05/24 Пнд 11:58:47 114928 93
>>114927
у нас в 10 классе так не принято
Аноним 13/05/24 Пнд 12:42:14 114929 94
>>114928
попробуй на кулинарном форуме спросить, как сварить вкусный суп, а потом поясни, что у вас в семье варить без говна не принято
Аноним 13/05/24 Пнд 13:14:53 114930 95
>>114929
ты видать охуенно высокого мнения о своём форуме
Аноним 13/05/24 Пнд 13:22:16 114931 96
>>114930
речь о тематике форума, а не о нём самом
Аноним 13/05/24 Пнд 13:35:10 114932 97
>>114926
Методом введения дополнительного угла
Аноним 13/05/24 Пнд 14:55:31 114934 98
>>114932
это для любых коэффициентов n и p сработает?
Аноним 13/05/24 Пнд 17:40:02 114937 99
>>29047 (OP)
В чём прикол инверсии? Ну типа я вижу её определение, но откуда оно взялось?
Аноним 13/05/24 Пнд 17:43:56 114939 100
Аноним 13/05/24 Пнд 17:44:16 114940 101
Аноним 13/05/24 Пнд 19:36:38 114945 102
>>114908
Вообще, достаточно херовая оптимизационная задача. Делить плохо, лучше бы ограничение выпуклое сделал. Если по существу, то у тебя решение одной легко переводится в решение другой. Но оно не гарантирует коллинеарности, ведь решений может быть не одно.
Аноним 14/05/24 Втр 00:48:25 114948 103
>>114945
Тогда меня интересует почему w_1 / \hat{\
gamma} и b_1 / \hat{\gamma} при оптимальных w_1 и b_1 будут решением (если будут) второй оптимизационной задачи?
Аноним 14/05/24 Втр 08:58:54 114950 104
>>114940
В геометрии, инверсия через окружности.
Аноним 14/05/24 Втр 14:30:40 114952 105
Помогите нубу. Пусть у нас есть короткая точная последовательность, которая не расщепляется. Есть ли какое-то понятие, которое характеризует степень этого нерасщепления? Я уже знаком с понятиями инъективного\проективного модуля, но как я понял это (необходимые) условия на суммируемые модули. А есть что-то именно про сам раскладываемый модуль?
Аноним 14/05/24 Втр 15:16:17 114953 106
>>114952
Твоя ктп расщепляется титтк соответствующий ей элемент в Ext^1 это 0. Берешь Hom(C,-) для своей ктп 0->A->B->C->0, пишешь длинную точную последовательность и смотришь куда переходит id из Hom(C,C) это и будет соответствующий элемент в Ext^1
Аноним 14/05/24 Втр 15:28:07 114954 107
>>114952
Допустим, у тебя есть фиксированные модули $A, B$ и ты хочешь узнать, можно ли подобрать X таким, что к.т.п. $0\to B\to X\to A\to 0$ не расщепляется и сколькими способами это можно сделать (по модулю изоморфизмов троек). Ответ на это даёт группа Ext1(A,B).
Аноним 14/05/24 Втр 15:28:32 114955 108
Бля ладно
Аноним 14/05/24 Втр 15:42:07 114956 109
>>114953
>>114954
Спасибо аноны, буду читать. Пока полистаю про расширения групп
Аноним 14/05/24 Втр 15:47:45 114957 110
>>114956
Насколько помню, теория с расширениями групп сложнее.
Аноним 14/05/24 Втр 17:10:56 114958 111
>>114957
Но с группами всё как-то нагляднее.. Что тогда лучше читать? Учебники по гомологической алгебре слишком сухие. Алюффи?
Вообще этим заинтересовался в контексте центральных расширений и их связи с проективными представлениями, так что в группы всё равно лезть придётся
Аноним 14/05/24 Втр 19:01:56 114960 112
>>114958
А, ну в проективных представлениях часто когомологиями групп интересуются, а они тоже через Ext-ы $\mathbb{Z}G$-модулей могут характеризоваться. Обычно весь материал даётся в специальных книжках по представлениям, но ничего конкретного я не посоветую.
Аноним 15/05/24 Срд 02:08:43 114965 113
Есть ли в природе нормальный "из аксиом" вывод преобразований Лоренца?

В качестве разминки интересен такой вопрос - если преобразование пространства переводит прямые в прямые, то оно обязательно должно быть линейным преобразованием. Это так? Вроде просто но чет не придумаю как доказать.
Аноним 15/05/24 Срд 09:11:47 114971 114
>>114965
ничего не знаю про преобразования Лоренца, но утверждение
>если преобразование пространства переводит прямые в прямые, то оно обязательно должно быть линейным преобразованием
безусловно неверное
Аноним 15/05/24 Срд 10:12:54 114972 115
>>114965
По-моему оно даже афинным не обязано быть
Аноним 15/05/24 Срд 11:32:58 114975 116
Я правильно понимаю, что чем "проще"/лучше кольцо (напр. коммутативное, без делителей нуля), тем беднее оно отражает структуру эндоморфизмов абелевых групп (если думать о модуле как о представлении кольца в кольце эндоморфзимов)?
Например в векторных пространствах можно представить только гомотетии, а если какое-то произвольное кольцо (скажем, с делителями нуля) - то можно представить что-то неиъективное вроде проекций.
Что погуглить, чтобы об этом побольше почитать?
Аноним 15/05/24 Срд 11:48:07 114977 117
Аноним 15/05/24 Срд 11:55:22 114978 118
>>114977
сделай линейную проекцию твоего пространства на любую прямую, а потом устрой на этой прямой любую самую уёбищную биекцию
Аноним 15/05/24 Срд 11:57:56 114979 119
>>114978
вертикальные прямые, правда, отобразятся в точку, но полагаю и это можно обойти
Аноним 15/05/24 Срд 12:02:15 114980 120
>>114979
нельзя: биективное отображение $V \to V$, сохраняющее прямые, необходимо афинное
Аноним 15/05/24 Срд 12:05:34 114981 121
Добавлю еще пару важных пунктов к >>114965
- преобразование взаимно-однозначное
- есть одна неподвижная точка

Собственно вот из за такой хуйни я и хочу найти вывод где все неявные предположения были бы явно перечислены в начале.
Аноним 15/05/24 Срд 14:18:01 114984 122
>>114981
теорема: всякая биекция $V \to V$, сохраняющая прямые, является афинным преобразованием. см. https://mathoverflow.net/questions/139978/line-preserving-bijection-of-mathbbrn-onto-itself

афинное преобразование, сохраняющее нулевой вектор, является линейным

таким образом, твоё преобразование должно быть линейным изоморфизмом
Аноним 15/05/24 Срд 22:55:05 114985 123
image.png 302Кб, 1067x741
1067x741
Что-то вообще не могу понять, как получили оценку корня.
Аноним 15/05/24 Срд 23:08:04 114986 124
>Я – такой неполноценный участник этого собрания. Полноценный сначала участвует в олимпиаде, а потом делается руководителем. Я никогда не был участником ни олимпиад, ни других состязательных соревнований в области математики и в других областях. Но я принимал участие в составлении задач (смотрите мою брошюру «О профессии математика»), в классификации способностей. Так что я – участник олимпиадного движения. Павел Сергеевич Александров сказал: «Если бы в моей молодости были олимпиады, я никогда не был бы математиком». Если олимпиадные способности есть, это хорошо, но если их нет, это не означает неспособности к математике. Тем не менее, накоплен материал, свидетельствующий, что большинство творческих математиков имеют отчасти и олимпиадные способности. Но всё же это два разных типа способностей, и поэтому победители олимпиад не должны зазнаваться, а неудачники не должны огорчаться.
Колмогоров.

Так что можете не расстраиваться, неудачники)
Аноним 15/05/24 Срд 23:59:07 114987 125
1638046694259.png 89Кб, 864x158
864x158
>>114986
Колмогоров вообще базовик
Источник каловый, но похуй
Аноним 16/05/24 Чтв 01:22:22 114988 126
Насколько сложно в аспу на матфак вышки поступить?
Аноним 16/05/24 Чтв 04:31:45 114989 127
>>114985
Мы проекцию на $V_1$ уже посчитали, так что длина проекции на $V_2$ ограничена $\left \Vert \xi - \langle \xi, \bar{h} \rangle \right \Vert$, где $h$ - это базисная функция $V_1$. Ну и понятно, что $(1-3.5)^2=(6-3.5)^2$, поэтому просто умножили на 2 внутри из-за лени видимо.

Очень интересный пример, думал книжка какая хорошая. Погуглил, а это какие-то сухие лекции, досадно.
Аноним 16/05/24 Чтв 05:00:08 114990 128
>>114987
Лол, ты думаешь, что он мечтал о "мире ... без женщин", потому что базовик? Анон, у него была другая причина...
Аноним 16/05/24 Чтв 11:27:47 114991 129
>>114989
>Ну и понятно, что $(1-3.5)^2=(6-3.5)^2$
Сука, я, блядь, несколько часов пытался разобраться, а они просто арифметику сократили. Спасибо, анон.
>Погуглил, а это какие-то сухие лекции, досадно.
В последнее время я интересные вещи только в таких заметках и нахожу.
Аноним 16/05/24 Чтв 11:29:22 114992 130
>>114948
Не будут. Их надо на гамму поделить. Тогда оно удовлетворяет твоим ограничениям из второй задачи. Допустим это будет не оптимум. Тогда у тебя есть $w_{2}$, на которых функционал лучше. Умножаешь их на гамму из первого решения, представляешь в первое, и получаешь противоречие с тем, что w1 это решение.
Аноним 16/05/24 Чтв 14:19:04 114994 131
>>114990
так именно эта причина и есть настоящая база...
Аноним 16/05/24 Чтв 14:44:50 114995 132
>>114990
Может они с Александровым просто дружили крепко. А может и пидоры. Хотя эту версию форсит какая-то еврейская девочка с шизой, хуярящая мужские гормоны.
В любом случае, бабы в 99% случаев абсолютно "бытовые" и мещанские существа. Они не могут в хорошем смысле "оторваться" от повседневной реальности. К подавляющему большинству мужчин это тоже относится
Аноним 16/05/24 Чтв 14:55:48 114996 133
>>114995
>Хотя эту версию форсит
чуть ли не сам Колмогоров рассказывал, как они неделями жили у Александрова на даче вдвоём, утром занимались математикой, вечером ходили на лыжах, а по ночам, что они делали, догадывайтесь сами чем

тут как бы и без форсов намёк очень жирный, хотя, конечно, ни разу не произносился прямо
Аноним 16/05/24 Чтв 15:01:09 114997 134
>>114996
>а по ночам, что они делали, догадывайтесь сами чем
Нет, не будем догадываться. Ты либо тащи железобетонные пруфы, либо манька промытая жидовскими сказочками что все все известные личности только и делали что в жопы поролись.
Аноним 16/05/24 Чтв 15:09:30 114998 135
>>114996
Я тоже у друга жил неделю вдвоём, днём гуляли, по вечерам аниме смотрели и чё?
Аноним 16/05/24 Чтв 15:11:15 114999 136
>>114997
никому в дейтсвтиельности не особенно интересно, поролись они на самом деле или нет, потому и пруфы никому не нужны

если тебе так очень важно, ты можешь с любой сколько угодно веровать в их жопную чистоту и даже защищать от посегательств на неё со стороны других анонов, вроде меня, и я даже оценю твои усилия по стремлению к истине, потому что мне, как и большинству людей, на это совершенно наплевать

>>114998
вы же там не занимались математикой, а после математики спортом. и так каждый день
Аноним 16/05/24 Чтв 15:37:19 115000 137
>>114999
>
>никому в дейтсвтиельности не особенно интересно, поролись они на самом деле или нет, потому и пруфы никому не нужны
>
>если тебе так очень важно, ты можешь с любой сколько угодно веровать в их жопную чистоту и даже защищать от посегательств на неё со стороны других анонов, вроде меня, и я даже оценю твои усилия по стремлению к истине, потому что мне, как и большинству людей, на это совершенно наплевать
Какие потешные повизгивания пойманной на гнилом пиздеже маньки.
Аноним 16/05/24 Чтв 15:47:56 115001 138
>>114995
> Хотя эту версию форсит какая-то еврейская девочка с шизой, хуярящая мужские гормоны.
Ну у тебя в манямирке если только.
Сразу видно, что ты даже близко к московской школе не подходил и не общался там ни с кем в 90ые-00ые. В таком случае целесообразно сидеть и помалкивать, а не ларпить мамкина борцуна с эсжовэ.
Аноним 16/05/24 Чтв 15:59:55 115002 139
>>114999
>вы же там не занимались математикой, а после математики спортом
Буквально так. Только математикой я один.
Аноним 16/05/24 Чтв 16:00:12 115003 140
>>115001
>московской школе
Что в хачевнике одна школа осталась? Анивей.
Пруфы будут?
Аноним 16/05/24 Чтв 16:01:55 115004 141
Вот бы найти куньчика, чтобы вместе под пледиком когомологии пучков вычислять.
Аноним 16/05/24 Чтв 16:31:07 115005 142
>>115001
> ты даже близко к московской школе не подходил и не общался там ни с кем в 90ые-00ые
Да и нахуй надо с пидерастами какими-то общаться
Аноним 16/05/24 Чтв 16:38:54 115006 143
>>115002
вот видишь
так ты не узнал настоящей любви, хотя она была близко

>>115000
ой, как ущемился-то

>>115004
увы
Аноним 16/05/24 Чтв 18:18:14 115007 144
В треде орудует тупая жируха-яойщица >>115006
Аноним 16/05/24 Чтв 19:47:40 115008 145
>>115007
не волнуйся, ты-то точно никому не нужен
Аноним 16/05/24 Чтв 20:10:29 115009 146
>>115008
Как же пиздятиной воняет.
Аноним 16/05/24 Чтв 20:21:50 115010 147
Аноним 16/05/24 Чтв 20:29:55 115011 148
Аноним 16/05/24 Чтв 21:00:37 115012 149
>>115005
гаразд би пидараси так адже москали
Аноним 16/05/24 Чтв 22:57:05 115013 150
Я вроде читал что Колмогорова отпиздили в подъезде, когда он уже стариком был.
А в молодости он сам прописал в ёбыч Лузину. Как-то не по-гейски...
Аноним 17/05/24 Птн 09:56:30 115015 151
>>29047 (OP)
Что у Зорича за заёб переписывать определения в логической форме значками?
Аноним 17/05/24 Птн 15:35:11 115016 152
Аноны помогите пожалуйста сравнить между собой 2 цены, не знаю как это сделать, наверняка это возможно, и тут еще вроде нужно окупаемость как то посчитать.
Дано:
1ая карточка которая стоит 210 монет, приносящая 2,85 монеты в час
2ая карточка которая стоит 119 монет и приносит 1,77 монеты в час.
Какую из них выгоднее купить?
Аноним 17/05/24 Птн 16:03:58 115017 153
больше года пробую решать егешные параметры, но справляюсь максиум в 10% случаев. У вас так же было, или это я один такой обрубок тупорылый? (лоб 27 уровня)
Аноним 17/05/24 Птн 16:05:04 115018 154
>>115016
Ты знаешь, что такое окупаемость?
Аноним 17/05/24 Птн 16:09:36 115019 155
>>115017
Ты решения смотришь, когда у тебя не получается? Свою проблему распознать не можешь? Что именно ты не умеешь: не знаешь свойства элементарных функций, не знаешь их графики, не умеешь слагаемые из одной части равенства в другую перекидывать, не замечаешь общие множители?
Аноним 17/05/24 Птн 16:23:00 115020 156
>>115019
ну чаще всего у меня не совпадает ответ из за ошибок в арифметике, иногда просто не понимаю как такое решать в принципе
Аноним 17/05/24 Птн 16:53:58 115021 157
>>115018
бля там с каждым новым лвлом этой карточки прибыль в час увеличивается не понятно на какую цифру, в этом проблема еще
Аноним 17/05/24 Птн 17:28:18 115023 158
Начинать лучше с алгебры или анализа?
Аноним 17/05/24 Птн 21:48:07 115024 159
Не раз слышал фразу, что физика занимается только дифференциальными уравнениями, не превышающими второго порядка. Почему так? Есть разъяснения? Мимо первокурсник, пока не было ещё курса ОДУ - извиняюсь, что пишу в /math/, но боюсь не получить корректного ответа в /sci/.
Аноним 18/05/24 Суб 00:57:31 115026 160
Аноним 18/05/24 Суб 00:58:13 115027 161
>>115023
закончишб алгебру потом анализ хДДД
Аноним 18/05/24 Суб 00:59:42 115028 162
>>115015
>Что у Зорича за заёб переписывать определения в логической форме значками?
так легче воспринимать большие формулировки
Аноним 18/05/24 Суб 01:09:54 115029 163
image.png 35Кб, 1331x1080
1331x1080
Аноним 18/05/24 Суб 01:35:26 115030 164
>>115021
Нормально задачу опиши, тогда, возможно, кто-то тебе и ответит.
Аноним 18/05/24 Суб 01:38:03 115031 165
>>115023
Абсолютно без разницы. Начни с того, что, как ты думаешь, тебе больше нравится.
Аноним 18/05/24 Суб 01:48:22 115032 166
Аноним 18/05/24 Суб 15:53:13 115034 167
>>115024
Занимаются. Просто в тех областях, которые не строятся на формализме Лагранжа и Гамильтона читай экзотика. Например в недифузионных процессах переноса возникают порядки, выше 2ого. Но их так-же можно свести к интегро-дифференциальным уравнениям, так что на полшишечки.
Аноним 18/05/24 Суб 18:11:42 115035 168
>>115023
Анализа, ведь он строился для рабочих и крестьян. А алгебра это что-то для ценителей.
Аноним 18/05/24 Суб 19:12:09 115036 169
1653566779653.jpg 40Кб, 739x568
739x568
image.png 198Кб, 2134x561
2134x561
Почему десмос просто добавляет к произведению по 0.4? Да и вольфрам тоже. Там же получается 1.4×1.28 и тд.
Причем если от функции отнять 1, то есть получится 0.4×0.28... , он начинает правильно считать.
Аноним 18/05/24 Суб 19:45:21 115037 170
>>115032
>>115034
Огромное спасибо за ответы, господа!
Аноним 18/05/24 Суб 20:40:07 115038 171
>>115036
Ебать я дебил, оно же правильно и считает, там же все кроме последнего числителя сокращается... Хуево быть тупым.
Аноним 19/05/24 Вск 06:39:04 115041 172
Есть разные способы установить взаимноднозначное соответствие между отрезком и прямой например с помощью функции арктангенс. Например введём равномерную случайную величину на отрезке от -pi/2 до pi/2, а затем спроектируем этот отрезок на всю числовую прямую с помощью функции арктангенс. Тогда у нас получится случайная величина заданная на всей числовой прямой. Но кажется что она уже не будет равномерно распределённой. Плотность распределения в области больших чисел будет меньше, чем вблизи нуля. А есть ли способ спроектировать отрезок напрямую так чтобы равномерно распределённое случайная величина на отрезке отобразилась на равномерно распределенную случайную величину на прямой? А если такого способа нет, то почему вообще разрешается рассматривать такие случайные величины, как равновероятный выбор одного вещественного числа из всего множества R? Когда выбираем из отрезка (0, 1) ещё как то можно представить, а из бесконечного интервала - как то не укладывается в голове... А может можно подобрать другую функцию распределения на отрезке -pi/2, pi/2, чтобы она в равномерную отображалась тем же арктангенсом?
Аноним 19/05/24 Вск 15:22:42 115043 173
>>115023
Смотря какая алгебра, линейная или общая. Общая интересная до теории Галуа, а дальше там идёт контент что выдумывали для док-ва ВТФ.
Анализ интересен, но именно в алгебраическо-топологической форме, до которой нужно год ползти.
Линейная алгебра это раздел анализа. Её буквально функциональщики придумали.
Аноним 19/05/24 Вск 16:15:45 115044 174
>>115043
>Линейная алгебра это раздел анализа
Бред.
Аноним 19/05/24 Вск 17:05:08 115047 175
>>115044
Бред то что линейная алгебра называется линейной алгеброй, а не линейным анализом или конечномерным анализом.
У Глазмана-Любича, кстати, называется правильно -Конечномерный линейный анализ.
У Дьедонне есть книжка об истории функционального анализа, там есть и о линейной алгебре. Так вот линейную алгебру в современном виде, какую можешь найти в нормальных учебниках Глазман-Любич, Халмош, Бурбаки, Булдырев-Павлов, Done Right/Wrong создали для потребностей функционального анализа. Всё что было до этого оказалось вообще не нужным, это нельзя было никак использовать.
Аноним 19/05/24 Вск 17:49:28 115048 176
В разъяснении парадокса монтиппайтона часто приводят пример, что если б ведущий открыл 98 из 100 дверей, то очевидно, что тебе нужно поменять дверь. А разве это корректный пример? А если б мне предложили открыть 98, а ведущий открыл одну пустую - я тоже должен поменять? А если 50 или 49 дверей? В оригинале задачи суть в том, что везде равное количество: тобой выбранных, открытых ведущим и предложенных на выбор.
Аноним 19/05/24 Вск 18:25:05 115049 177
>>115043
>Её буквально функциональщики придумали.
Тоже бред. Достал уже везде к линалу функан тащить. У тебя травма, что ли? Тебя в функан-петуха прописать?
Аноним 19/05/24 Вск 20:47:35 115051 178
image.png 51Кб, 975x243
975x243
image.png 67Кб, 986x343
986x343
>>115047
Функанщики любят линал, потому что любят операторы, очевидно. Поэтому и книжки пишут соответствующие (я ещё на первом курсе заметил, насколько Халмош отличается от другого изложения). В твоём Глазмане-Любиче как раз говорится, что это "изложение линейно алгебры с точки зрения функционального анализа", а не сказано, что это раздел функана.
>Так вот линейную алгебру в современном виде, какую можешь найти в нормальных учебниках Глазман-Любич, Халмош, Бурбаки, Булдырев-Павлов, Done Right/Wrong создали для потребностей функционального анализа. Всё что было до этого оказалось вообще не нужным, это нельзя было никак использовать.
Я почитал, что там написано, и мб не нашёл нужный момент, но там говорится лишь о том, что взгляд на линейную алгебру, который мог бы дать идеи для обобщения (т.е. функан как будто бы как раз рассматривается как анализ, к которому применяют методы из линала, а не наоборот), появился только в начале 20-го. Ты из этого какие-то не сильно обоснованные импликации делаешь.
Более того, формулировка линейной алгебры как чего-то, что происходит в контексте векторного пространства, это как раз скорее "общеалгебраизация". И в любом учебнике по общей алгебре это просто, ну, видно, что изложение линала никак не выбивается из общего нарратива изучения алгебраических структур.
Собственно, только последнее предложение относится к посту с вопросом: изучение линала и изучение основ мат.анализа это совершенно непохожие деятельности.
Аноним 19/05/24 Вск 20:50:07 115052 179
>>115048
У тебя одна и та же математическая модель для разного количества. Количественный ответ в соответствии с ней ты можешь сам тогда посчитать.
Аноним 19/05/24 Вск 23:51:13 115056 180
122305c2d252c33[...].jpg 15Кб, 236x314
236x314
Привет. Посоветуйте, пожалуйста, годных учебников по формальной логике и теории алгоритмов. Сильно заинтересовала тема формальной верификации, теория типов, PLT и вот в целом вся эта движуха на стыке информатики и логики.
Из бэкграунда - есть некоторая база в автоматах и вычислимости, отличу общерекурсивную функцию от частично рекурсивной, теорему Клини какую-нибудь сформулирую и докажу. Но всё это скорее разрозненные знания, поэтому хотелось бы последовательно структурировать это всё и в целом интересно это стало. Спасибо.
Аноним 20/05/24 Пнд 00:03:43 115057 181
>>115056
Конечно, советую: Homotopy Type Theory: Univalent Foundations of Mathematics.
Тут такое лучше не спрашивать, вопрос очень косвенно с математикой связан, погугли лучше сам.
Аноним 20/05/24 Пнд 00:07:27 115058 182
Аноним 20/05/24 Пнд 00:08:59 115059 183
>>115057
Ага, а верификацию он на Arend-е делать будет.
>>115056
Не верь ему, это оторвет тебе жопу.
Аноним 20/05/24 Пнд 01:33:39 115060 184
>>115058
Пидор какой то. В письтех спецом что ли таких отбирают?
Аноним 20/05/24 Пнд 02:49:14 115061 185
>>115058
>Но тут только интуиционистская логика
Разве? Это вроде бы просто принято так в cockque-сообществе, а так там вполне можно прописать в аксиомах и закон исключённого третьего.
мимо не разбираюсь но что-то такое от логиков слышал
Аноним 20/05/24 Пнд 08:26:07 115062 186
>>115061
Да, никто не мешает дописать исключённое третье.
(А вот модальную логику, например, только в моделях уже или deep embedding.)
Аноним 20/05/24 Пнд 17:24:54 115067 187
Screenshot2024-[...].jpg 119Кб, 713x394
713x394
Screenshot2024-[...].jpg 130Кб, 636x817
636x817
Признак сходимости на пикриле выдаётся за признак сходимости Гаусса, но в Википедии и учебниках признак Гаусса по другому формулируется (пик2). Это действительно эквивалентная формулировка?
Аноним 20/05/24 Пнд 21:25:43 115068 188
Как правильно визуализировать цепи, циклы, генераторы гомологий, чтобы это не просто интуитивно было, но и формально?
Меня, собственно, одномерные интересуют. Понятно, что на торе, допустим, есть стандартная клеточная структура, можно посчитать и понять, что меридиан и широта будут генераторами гомологий. С триангуляцией так просто не получится уже, там много треугольничков. С сингулярными цепями на первый взгляд всё ещё хуже.
Но я видел, как эту интуицию к любым петлям применяли, утверждая, например, что если петля ограничивает открытый диск, то она гомологична нулю. Эта интуиция откуда берётся? Из частного случая теоремы Гуревича об изоморфизме абелинизации фундаментальной группы и первых гомологий? Или из более фундаментальных соображений, мб как-то сингулярные гомологии визуализируют?
Аноним 20/05/24 Пнд 21:33:27 115069 189
>>115068
>мб как-то сингулярные гомологии визуализируют?
Например, можно разбить границу такой петли на три "дуги" (представить петлю как сумму трёх ориентированных в одну сторону путей) и внутрь три "треугольника" впихнуть так, чтобы "рёбра" имели противоположную ориентацию как образы соответствующих сингулярных 2-цепей, тогда петля будет границей суммы этих "треугольников".
Аноним 20/05/24 Пнд 22:07:14 115070 190
>>115069
Разбиение я такое делаю, так как граничный оператор знаки альтернирует. Но мб как-то можно и без этого обойтись?
Аноним 21/05/24 Втр 00:17:04 115071 191
>>115070
А, ну реально можно. Я по сути повторяю (условно говоря, возможно) обычную триангуляцию и не пользуюсь преимуществом сингулярных гомологий, что можно грани в одну точку схлопывать. Тогда петлю можно будет легко как границу реализовать.
Аноним 21/05/24 Втр 05:02:49 115072 192
>>115051
>потому что любят операторы, очевидно
До них операторов не было в лин. алгебре. Не было ни операторов, ни двойственных пространств, ни функционалов. Даже векторных пространств не было. Нихуя не было, потому что не было причин эти вещи рассматривать с тч зрения алгебры-геометрии. Были только координатное представление векторов, определители, векторное и скалярное произведение. Короче всё то, что сейчас существует в книжках по аналитической геометрии, которая уже давно история.
>а не сказано, что это раздел функана
Потому что другой точки зрения не существует. 90% контента линала придумали функанщики. Даже такие вещи как вообще определения вект. пространства.
>И в любом учебнике по общей алгебре это просто, ну, видно, что изложение линала никак не выбивается из общего нарратива изучения алгебраических структур.
А ты не думал, что это линал повлиял на алгебру, а не наоборот? Современную алгебру создали Артин с Нётер. Артин любил использовать линал.
Аноним 21/05/24 Втр 06:41:28 115073 193
>>115072
>Не было ни операторов, ни двойственных пространств, ни функционалов.
Ну да, я и говорю, что операторное изложение линала это призак автора-функанщика. Но не все изложения линала такие. И двойственным пространствам и линейным формам уделяется в алгебраическом курсе довольно мало времени.
Зато любят очень билинейные формы, а их ещё Гаусс любил и изучал, правда в другом контексте. А ещё линал это приведение матриц к "каноническим" формам, чем Грассман и Жордан занимались.
Векторные пространства как вещь появились поздно, но их не функанщики строго определили, а Пеано по трудам Грассмана. Функанщики это определение сделали намного более популярным, да, и множество результатов именно они доказали.
> 90% контента линала придумали функанщики
Ну это мем.

>А ты не думал, что это линал повлиял на алгебру, а не наоборот?
Ого, теории колец и алгебр у нас тоже теперь на плечах функана стоят? Чудеса!
>Современную алгебру создали Артин с Нётер.
Половину современных алгебраических структур определил ещё Дедекинд на поколение раньше.
Аноним 21/05/24 Втр 08:47:27 115074 194
>>115072
Твои оценки могут сработать, если изначально определить линал как конечномерный случай функана. Тогда да, количество результатов, полученных в функане, заметно больше, полагаю, чем для конечномерных пространств. Проблема только в том, что большинство результатов в том, что мы обычно подразумеваем под названием линейная алгебра (классический набор теорем), получена алгебраистами алгебраическими методами. Поэтому логичнее смотреть на функан как на обобщение.
Аноним 21/05/24 Втр 08:58:50 115075 195
>>115073
Пиздец, ну давай гомоморфизмы из теории групп выкинем, ради "правильного изложения"? А заодно и сами группы, они поздно появились? Заебал петух
Аноним 21/05/24 Втр 09:55:09 115076 196
>>115072
>Потому что другой точки зрения не существует.
Конечно существует. Например, можно рассматривать линейную алгебру как "К-теорию над точкой", и проективные модули и векторные расслоения как естественное обобщение векторных пространств и их геометрии.
Собственно, стандартный курс линейной алгебры сейчас выглядит именно так, как он выглядит, потому что это минимальный набор определений/фактов, который может служить отправной точкой как для любой теоретической точки зрения/обобщения, так и для конкретных приложений.
Аноним 21/05/24 Втр 09:57:25 115077 197
>>115075
Ты о чём вообще, болезный? Можешь мысли когерентно выражать?
Аноним 21/05/24 Втр 10:16:41 115078 198
>>115073
>Но не все изложения линала такие.
А какие есть ещё? Я знаю только два, координатный ад с смешанными произведениями из 19 века, который остался только в методичках по линалу в провинциальных вузах, и нормальный, как конечномерный функан. Причём функан оверкилл, линал хорошо мотивируется обычным анализом.
>Зато любят очень билинейные формы, а их ещё Гаусс любил и изучал, правда в другом контексте.
Ты линал по учебникам 19 века изучал?
>а Пеано по трудам Грассмана
Это всё осталось незамеченным по причине того что никто особо не видел пользу от этого всего. Никто не знал как это приложить к другим областям. У Грассмана были попытки, в ответ на доёбы Мёбиуса, но я их не изучал.
>Половину современных алгебраических структур определил ещё Дедекинд на поколение раньше.
А Архимед изобрёл интегрирование.

>линал хорошо мотивируется обычным анализом
Изучаем "наивный анализ", можно даже без общих определений, а на примере какой-нибудь одной конкретной функции, например $e^x$, которую можно определить из чисто инженерных соображений быстрого счёта. Так студент притрагивается к дифференциированию, интегрированию и ряду тейлора.
Дальше очень быстро можно рассказать, что всё что мы придумали для многомерного анализа не очень то и работает и получается какой-то символьный координатный хаос с модулям.
Дифференциируя мы находим линейную апроксимацию функции в точке. И потому не плохо бы эти линейные функции, а для многомерного случая операторы, изучить, а так же и то, где они действуют, на векторных пространствах. И так начинаем учить линал, как инструмент для анализа. Потому что мотивация в виде систем уравнений идиотская, никому не нужны системы линейных уравнений, просто сложно придумать хорошую задачу, где потребовалось бы их решать, кроме как по приколу.
Аноним 21/05/24 Втр 10:20:00 115079 199
>>115074
>что большинство результатов в том, что мы обычно подразумеваем под названием линейная алгебра (классический набор теорем), получена алгебраистами алгебраическими методами
Какие например? Кронекер-Капели и ЖНФ?
>>115076
Я имею ввиду точки зрения для студентов.
Аноним 21/05/24 Втр 10:40:15 115080 200
>>115077
Это не ты функан-петух? Прости, если обознался
Аноним 21/05/24 Втр 13:06:29 115082 201
Так смешно читать высеры третьекурсника-мамкиного первокультурщика про линал. Примерно также, как и высеры функанщика.
Аноним 21/05/24 Втр 13:42:46 115084 202
>>115082
Наиболее смешно, когда кто-то врывается в дискуссию, только лишь чтобы назвать всех дураками, но не предлагает ничего от себя

Этакая потешная попытка самоутвердиться

мимо не участвовал
Аноним 21/05/24 Втр 13:55:05 115085 203
>>115082
Я уже писал, возьми книгу по истории функана от Дьедоне, где он пишет, что всё что было до начала 20 века это координатный ад, который ни для чего, кроме решения систем лин. уравнений, не годился. И пришлось придумывать всё с нуля. Получилась новая область, которая с некоторыми оговорками поглощала весь линал что был до.
И я вот понимаю, хотя может и не смогу выразить точно словами, важность анализа. Но важность конечных систем лин. уравнений я понять не могу. Может расскажешь, на кой хуй нам их решать, кроме как просто так? В курсе алгебры какие-нибудь лупы Мафанг, колеса и пр. даже не упоминают, потому что они в других разделах математики, которые будет учить студент, не пригождаются. Почему бы так не поступить с системами лин. уравнений, и в качестве цели лин алгебры обозначить её как прибор для анализа, и вообще называть линейным анализом, а не алгеброй? При таком подходе куча определений становятся логичными. Скажем с алгебраической точки зрения рассматривать двойственные пространства нет никакого смысла, а без них в общем никак, потому что отождествление пространства с дважды двойственным к нему даёт много свободы с обращениями с векторами. С точки зрения анализа рассматривать функционалы это само собой разумеющееся действие.
И заметь, я говорю не о функане, а о многомерном анализе. Если смотреть стандартную программу, то линал изучается просто без какой-либо цели, отчего изучать его тупо не интересно, нужно быть аутистом любителем логик, аксиом теории множеств и подобных вещей, чтобы сохранять мотивацию. И лишь в 2-3 семестре линал используют для анализа. Почему бы сразу это не обозначить и не дать пару примеров, а не изучать как вещь в себе, я не понимаю.
Аноним 21/05/24 Втр 16:16:42 115092 204
>>115078
Ты Ленга не открывал? Винберга не видел?
>Ты линал по учебникам 19 века изучал?
Напоминаю основной список тем: линейные уравнения, теоремы о базисах, всякие rank-nullity теоремы, приведение матриц к каноническим формам (сюда же спектральная теорема для симметрических матриц, которая ещё Коши доказана была) и некоторые прочие матричные разложения, приведение билинейных/квадратичных форм к каноническому виду, критерии положительности/отрицательности, приведение билинейных/квадратичных форм на евклидовом пространстве, тензорная алгебра векторных пространств.
Просвети меня, что из этого для конечномерного случая тоже впервые было доказана функанщиками. Или какие очень важные темы (не в твоём понимании, а в соответствии с тем, что изучают) я забыл.

>А Архимед изобрёл интегрирование.
Смешное передёргивание, но я вроде специально написал "современных".
Аноним 21/05/24 Втр 16:17:56 115093 205
>>115085
А важность систем полиномиальных уравнений тоже понять не можешь?
Аноним 21/05/24 Втр 16:29:37 115094 206
>>115092
Алсо частый метод доказательств на комплементарные разложения пространств (что в линале, что в конечномерной теории представлений) это индукция.
Аноним 21/05/24 Втр 16:56:57 115095 207
если под "геометрией модулей" вы тут понимаете серра-свана, то это значит, что понимание математики тут на уровне читателя вербитоблога (то есть никакое)
Аноним 21/05/24 Втр 16:59:22 115096 208
>>115029
Это что? что с этим мне делать?
Аноним 21/05/24 Втр 17:12:43 115097 209
>>115095
Я думал, геометрия модулей это геометрия на пространствах модулей
Аноним 21/05/24 Втр 20:36:34 115098 210
>>115097
Но тогда аналогия с векторными пространствами идёт по пизде
Аноним 21/05/24 Втр 21:22:44 115099 211
Как решать тригонометрические уравнения?
Аноним 21/05/24 Втр 21:29:02 115100 212
>>115099
Это байт на тред про экспоненту?
Аноним 21/05/24 Втр 21:34:48 115101 213
>>115100
Да нет..
Тригонометрическую окружность выучил, формулы приведения и другие тригонометрические формулы тоже выучил.
Однако смотрю на эти уравнения, но пока их не понимаю. Функции начать учить?
Аноним 21/05/24 Втр 22:18:10 115102 214
>>115101
При чём тут другие функции? И что конкретно тебе непонятно? Мы тебе в голову вряд ли залезем.
Аноним 21/05/24 Втр 22:56:00 115104 215
n человек скинулись и купили акции на S1 рублей. Акция выросла в цене и теперь стоит S2 > S1 рублей. i ный человек скинул pi рублей. Нет ли подводных посчитать, что каждый в итоге должен получить S2*pi//S1, где // - целочисленное деление. Не будет ли недоразумений, кроме того, что какие то копейки останутся на поделенными?
Аноним 22/05/24 Срд 00:07:14 115106 216
Произведение бесконечно малых есть бесконечно малая высшего порядка чем оба множителя?
Аноним 22/05/24 Срд 04:48:16 115108 217
>>115106
Докажи по определению, полезно.
Но вообще да.
Аноним 22/05/24 Срд 13:38:59 115110 218
>>115104
Если делить по "пацански" то пропорционально начальному вкладу, т.е выплата $i$-ому участнику должна составить $p_i\frac{S_{2}}{S_{1}}$. Т.е. как будто бы он сам покупал эти акции. А так делить нужно так, как договаривались изначально.
Аноним 22/05/24 Срд 13:40:39 115111 219
>>115104
А, стоп, откуда вообще целочисленное деление пошло?
Аноним 22/05/24 Срд 14:34:31 115112 220
Скожите, почему так вышло, что синус будет длинрй противолежащего катета, а косинус длиной прилежащего катета на круге.
Аноним 22/05/24 Срд 14:42:08 115113 221
>>115112
И еще скажите как меняется решение проблемы при кол-ве цифер в пи. Типа есть ли разница в том, чтоб решать что-то с 6 или 3 мя. Какая разница?
Аноним 22/05/24 Срд 14:44:35 115114 222
>>115112
Потому что гипотенуза это радиус, а радиус 1
Аноним 22/05/24 Срд 15:20:04 115115 223
>>115113
>Типа есть ли разница в том, чтоб решать что-то с 6 или 3 мя. Какая разница?
Есть. Ошибки копятся при вычислениях. $1.8$ это почти $2$, разница $0.2$. $2+2=4$, $1.8+1.8=3.6$, разница уже $0.6$.
Аноним 22/05/24 Срд 15:45:41 115116 224
Посоветуйте задачники по абстрактной алгебре с решениями.
Аноним 22/05/24 Срд 18:43:00 115118 225
>>115116
атья-макдональд

>с решениями.
сюда тащи
Аноним 22/05/24 Срд 18:46:22 115119 226
>>115110
>>115111

Я так и записал. Вопрос как копейки выкинуть. Я же ирл не могу выплатить 7/3 рублей. Поэтому я и спрашиваю, если я сначала умножу p_i на S2, а потом нацело поделю на S1, все ок будет? По модулю исчезнувших копеек. Нацело, имееется ввиду, что это не вещественные рубли, а целочисленные копейки
Аноним 22/05/24 Срд 22:16:38 115120 227
>>115104
>>115119
Просто округляй самым последним действием, вычисления все проводи с наивысшей возможной точностью. Не нужно нацело ничего делить.
Аноним 22/05/24 Срд 22:19:08 115121 228
>>115119
А тебе точно интересен вопрос копеек? По-хорошему при выплатах ты должен округлять копейки вверх, но можешь договориться и до округления вниз с последующем реинвестированием. Или брать комиссию, чтоб мочь с нее выплатить округленные вверх копейки. Но это все пустые и душные формальности. Нормальные люди инвестируют заметно больше 1 рубля.
Аноним 22/05/24 Срд 23:02:50 115122 229
>>115118
>сюда тащи
Чтобы мелкоблядь насрала тыщу постов про то как все можно решить преобразованием Фурье. Не рекомендую.
Аноним 22/05/24 Срд 23:45:38 115123 230
>>115122
Ну так преобразование Фурье реально имба, которая очень помогает почти везде и почти всегда.
Аноним 22/05/24 Срд 23:51:18 115124 231
>>115121
Да. Не так ценны копейки, как учет всего с точностью до копейки

> Нормальные люди инвестируют заметно больше 1 рубля.
Да хоть миллиард. Все равно не поделится нормально и пару копеек придется отбросить
Аноним 23/05/24 Чтв 06:36:56 115126 232
Что читать/решать, чтобы взять место на всеросе по математике и при этом не зафаршмачиться о "не настоящую математику"? Что одобряют пучкисты и их вождь? Я в 11 перехожу, если что.
Аноним 23/05/24 Чтв 06:38:24 115127 233
>>115122
>школотрон не слышал про двойственность Понтрягина
Аноним 23/05/24 Чтв 11:50:52 115128 234
Объясните, пожалуйста.
Тема: "Однородные тригонометрические уравнения"

В учебнике пишут: "Итак, дано уравнение
a sinx + b cosx = 0, где а≠0, b≠0.
Разделив обе части уравнения почленно на cos x, получим:

(a sinx / cosx) + (b cosx/cosx) = 0/cosx
т.е. a tgx + b = 0, в итоге приходим к простейшему тригонометрическому уравнению

tgx = - (b/a)"

Короче, не понимаю, почему почленно делят, я думал, что можно только так разделить: (a sinx + b cosx) / cosx = 0 / cosx
Аноним 23/05/24 Чтв 12:08:46 115129 235
>>115128
А в чем разница? Вот например (2+4)/3= 2/3+4/3. Можно сразу весь числитель разделить на знаменатель, а можно почленно, получится одно и то же.
Аноним 23/05/24 Чтв 13:07:17 115130 236
>>115127
>шизы не слышат мои голоса в голове
Аноним 23/05/24 Чтв 13:58:53 115131 237
>>115122
смотрю, в петушиной жопе теперь горит вечный огонь
Аноним 23/05/24 Чтв 14:37:33 115132 238
>>115131
Убей себя, чмошница ущербная.
Аноним 23/05/24 Чтв 15:11:15 115135 239
>>115132
ещё немного, и на тебе можно будет проводить управляемый термоядерный синтез

токамак-петух!
Аноним 23/05/24 Чтв 15:38:25 115137 240
>>115135
Я тебя в рот ебал.
Аноним 23/05/24 Чтв 15:47:08 115138 241
Аноним 23/05/24 Чтв 16:53:57 115139 242
>>115138
Давай, насри еще тысячу постов, мне все равно делать нехуй.
Аноним 23/05/24 Чтв 17:28:44 115140 243
>>115139
понимаю - обычное состояние для неосилятора
Аноним 23/05/24 Чтв 18:08:18 115141 244
>>115140
Ну тебе виднее. Неосилятор то тут только ты.
Аноним 23/05/24 Чтв 18:11:57 115142 245
>>115141
горящий петух всё ещё в стадии отрицания
Аноним 23/05/24 Чтв 18:55:30 115143 246
>>115142
Это ведь ты НМУ-петух который засрал весь раздел о том как охуенно в НМУ и нет ничего лучше в жизни чем решать задачки НО не пошел на экзамены потому что "были другие приоритеты"?
Аноним 23/05/24 Чтв 19:22:48 115144 247
>>115143
тебе пора бы вылазить уже из мира фантазий и наконец начать адекватно принимать реальность

А то так и останешься термоядерным неосилятором
Аноним 23/05/24 Чтв 20:20:46 115145 248
>>115144
Ну значит точно тот самый петух проецирует про неосиляторство и горящую сраку. Не удивительный поворот.
Аноним 23/05/24 Чтв 22:23:21 115148 249
Почему $d^2x = 0$, если $x$ — это независимая переменная?
Аноним 23/05/24 Чтв 22:38:41 115149 250
>>115148
подробнее сформулируй вопрос
Аноним 23/05/24 Чтв 22:44:04 115150 251
image.png 135Кб, 821x419
821x419
>>115149
Какого рода подробности нужны?
Читаю Фихтенгольца, не понимаю то, что написано внизу скриншота.
Аноним 23/05/24 Чтв 22:48:47 115151 252
>>115150
видимо, имеется в виду, что если $x$ не зависит от $t$, то $x'_t = 0$, тем самым дифференциал нулевой
Аноним 23/05/24 Чтв 23:34:21 115152 253
>>115148
Независимая = (в этом контексте) не зависит от переменной, по которой дифференцируем.
Аноним 23/05/24 Чтв 23:43:19 115153 254
>>115128
во-первых, конечно, чтобы делить на [math]\quad \cos x \quad[/math], он должен быть ненулевым: [math]\quad \cos x \neq 0 \quad[/math], откуда получаем, что ни для одного целого k x не имеет вид [math]\pi / 2 + k \pi[/math].
>я думал, что можно только так разделить: (a sinx + b cosx) / cosx = 0 / cosx
вообще, по теореме, да. можно поделить обе части равенства и получить равносильное равенство. на этом основан первый шаг преобразования.
еще деление дистрибутивно справа: [math](p + q) / r = p / r + q / r[/math]. это второй шаг преобразования.
видимо, в учебнике мелкие шаги пропущены, т. к. нужные теоремы уже хорошо известны и читатель легко разберется.
Аноним 24/05/24 Птн 00:22:39 115155 255
image.png 42Кб, 871x119
871x119
>>115148
Дифференциал является функцией двух переменных: точки, в которой мы его вычисляем, и приращения. Тогда производная от приращения, которая считается независимой от x, тривиальным образом зануляется. Помедетируй над этим, можешь $dx$ обозначать $a$, например, чтобы сделать акцент на этом, но в целом это довольно неуклюжее соглашение.
Аноним 24/05/24 Птн 08:48:16 115156 256
Если любые линейные пространства равной размерности изоморфны друг другу, то получается, что любое линейное преобразование из одного пространства в другое можно представлять как оператор на одном пространстве?

Ну то есть если есть пространства многочленов U и V размерности 3 и 2 и преобразование f : U -> V, то его можно представить, как оператор над $\mathbb{R}^3$, у которого просто одно измерение сжимается в 0?
Аноним 24/05/24 Птн 12:04:56 115157 257
>>115156
Полагаю, что да.
Пусть у тебя есть $f:U_1\to U_2,\ \operatorname{dim}{U_i}=k_i$, тогда они изоморфны $\mathbb{R}^{k_i}$, их можно вложить в $\mathbb{R}^n,\ n\ \geq \operatorname{max}_i{k_i}$, при этом можно расщепить $\mathbb{R}^n\cong\mathbb{R}^{k_i} \bigoplus \mathbb{R}^{n-k_i}$, и задать полное отображение на всех парах слагаемых так, чтобы на $\mathbb{R}^{k_1}\to\mathbb{R}^{k_2}$ совпадало как раз с изначальным $f$ (ну только ещё с изоморфизмами отождествления с $\mathbb{R}^{k_i}$ взять композицию), на остальных можно занулить или как-то иначе определить. Если $n$ выбрать ровно максимумом, то тогда с одной стороны будет только одно слагаемое, тогда на одной паре будет наше изначальное отображение, а на второй как захочешь (ну только чтобы оно линейным было офк). В принципе, это как раз то, что ты написал.
Аноним 24/05/24 Птн 12:30:45 115158 258
>>115157
после "расщепить" ничего не понял, но спасибо)
Аноним 24/05/24 Птн 13:10:17 115159 259
>>115156
>то получается, что любое линейное преобразование из одного пространства в другое можно представлять как оператор на одном пространстве
да; с точки зрения координат это просто заполнение любой матрицы до квадратной
Аноним 24/05/24 Птн 14:15:53 115160 260
>>115158
Ну, ручками это делается так, что ты, например, вкладываешь плоскость в пространство, тогда всё пространство это прямая сумма плоскости и прямой, не лежащей в ней. Для векторных пространств так всегда сделать можно, для более общих модулей уже нет. Это равносильно тому факту, что если ты выбираешь какое-то подпространство, то всё пространство разлагается в прямую сумму подпространства и дополнения к нему (можешь погуглить split exact sequence, если интересно).
Я в более общем виде отметил ещё тот факт, что если у нас есть прямая сумма векторных пространств (на самом деле и модулей) $\bigoplus_i V_i$, то задать отображение $F:\bigoplus_i V_i\to \bigoplus_i V_i$ это то же самое, что задать семейство отображений между всеми парами слагаемых $\{f_{ij}:V_i\to V_j\}$
Аноним 24/05/24 Птн 15:18:17 115161 261
1612242239671.png 170Кб, 919x763
919x763
Почему в коммьюнити математиков так много пидорасни?
Аутистам ебаться хочется, но тёлки не дают? Ну не друг друга же жарить...
Аноним 24/05/24 Птн 15:27:23 115162 262
Существуют ли теоремы, начинающиеся с квантора существования? А то пока что кажется, что они все начинаются с "для любого..."
Аноним 24/05/24 Птн 15:52:28 115163 263
>>115161
тут есть несколько аспектов

1) указанный герой в первую очередь школьный учитель, а не математик. школьные учителя с высокой вероятностью педофилы, потому что они искренно любят детей

2) гомосексуализм - это более высокая форма любви, чем межполовая любовь. разливаться на этот счёт я не буду, но это более-менее ясно всякому культурно развитому человеку (если он помыслит над этим вопросом здраво, отодвинув штампы и догмы). математики - люди культурные и развитые

3) большая часть выдающихся математиков счастливы в браке и имеют множество детей. с этим у них обычно всё в порядке
Аноним 24/05/24 Птн 15:56:05 115164 264
>>115161
Потому что "пидорство" это на самом деле чрезвычайно редкая форма девиации на далеком цатом месте в списке других возможных девиации после - на вскидку - асексуальности, бисексуальности, то-чего-нельзя-называть, люди трахающие деревья... Было зафорсено ЖИДАМИ для своих непонятных целей. Так же как сейчас в прямом эфире можно наблюдать за форсом транс-поебистики, когда каждый первый подросток считает себя небинарной нетакусей, когда на самом деле это сверх-редкие отклонения возникающие у одного человека на миллионы.
Аноним 24/05/24 Птн 18:20:50 115165 265
>>115160
Отмечу, что суммы тут конечные подразумеваются.
Аноним 24/05/24 Птн 20:19:37 115166 266
Аноним 25/05/24 Суб 01:25:21 115167 267
>>115163
> 2)
Если это назвать дружбой, и не сосать хуи друг другу, то во всём согласен
Аноним 25/05/24 Суб 11:01:50 115168 268
>>115167
Хватит спорить с ядром европейской классической философии.
Аноним 25/05/24 Суб 14:19:23 115169 269
>>115161
Потому что потребность в межпоколенческом трансфере объемных и комплексных знаний неизбежно приводит к возникновению менторства, близких личных отношений между учителем и учеником и легальной или полулегальной институциализации педофилии. Это антропологическая константа. Античная философия, восточные учения и единоборства, христианская церковь, частные английские пансионы, императорское училище правоведения, физико-математические спецшколы, система ДЮСШ и олимпийский резерв, балетные, музыкальные и театральные школы, все, что аффилировано с гнесинкой, гитисом и репинкой... - и так далее, до бесконечности.

Все олимпиадное движение (не только в РФ, но и везде) снизу доверху переполнено педофилами. Вся система создана и поддерживается педофильским лобби. Это секрет полишинеля. В пользу физмат-аспергеров можно сказать, что в этой среде явные эксцессы происходят довольно редко и большинство отношений останавливается на платонической стадии. А вот в каком-нибудь балете или музыкалке просто берут и ебут - причем с прямого согласия родителей, которые сами в теме. Если в среднем по популяции 25% девочек и 5% мальчиков подвергаются сексуальной агрессии или насилию - то можешь себе представить, что происходит внутри СПЕЦИАЛИЗИРОВАННЫХ в этом плане структур.
Аноним 25/05/24 Суб 14:44:20 115171 270
>>115161
Так это московские чуханы. Гротендик зрелых баб ебал.
Аноним 25/05/24 Суб 16:51:54 115172 271
>>115162
>Существуют ли теоремы, начинающиеся с квантора существования?
тебя интересует существование просто формально выводимых теорем подобного вида [math]\exists x A[/math] или именно каких-либо замечательных теорем? первые, конечно же, существуют. последние тоже, например, существование модели какой-либо конкретной теории, чем доказывается ее непротиворечивость.
>А то пока что кажется, что они все начинаются с "для любого..."
так это потому что, как правило, замечательная теорема [math]A[/math] представляет собой обобщенное утверждение о целом круге похожих между собой частных явлений, выражаемых формулами [math]A_1, A_2, ...[/math] и т. д. [math]A_x[a_i][/math] получается подстановкой значения [math]a_i[/math] в [math]A[/math] вместо [math]x[/math]. получается: "для любого [math]x[/math] выполняется [math]A[/math]"
вообще, для любых формулы [math]A[/math] и переменной [math]x[/math] из [math]A[/math] можно вывести [math]\forall x A[/math], и наоборот, из [math]\forall x A[/math] выводима [math]A[/math]. поэтому не имеет значения, формулировать ли теорему [math]A[/math] со вхождением в нее свободных переменных [math]x, y [/math] или же связывать их квантором всеобщности: [math]/forall x /forall y A[/math] - обе формулировки равносильны друг другу.
Аноним 25/05/24 Суб 16:54:18 115173 272
>>115164
все люди имеют право свободно выбирать гендер
навязывание гендера является нарушением права человека
Аноним 25/05/24 Суб 16:59:51 115174 273
>>115172
>или же связывать их квантором всеобщности: [math]/forall x /forall y A[/math]
Аноним 25/05/24 Суб 17:02:23 115175 274
>>115172
>или же связывать их квантором всеобщности: [math]\forall x \forall y A[/math]
Аноним 25/05/24 Суб 19:58:54 115179 275
>>115172
Что такое замечательные теоремы?

> первые, конечно же, существуют. последние тоже
Можешь привести конкретные примеры?
Аноним 25/05/24 Суб 20:46:06 115180 276
>>115179
>Что такое замечательные теоремы?
теоремы, гласящие о решении замечательных проблем. например [math]x = x[/math], [math]( A \to B ) \& ( B \to C ) \to ( A \to C ) [/math], [math]2 + 2 = 4[/math] - это не замечательные теоремы.
а указание Больцано - на существование всюду непрерывной всюду недифференцируемой функции, Дирихле - на существование всюду разрывной функции - этотзамечательные теоремы.
Аноним 26/05/24 Вск 13:07:54 115181 277
IMG202405261242[...].jpg 13Кб, 171x162
171x162
Когда-то давно в какой-то книжке или может журнале видел примерно такую задачу. Есть круглый барабан по типу Поля чудес и на нем стоят 5 одинаковых ящиков (или может в нем сделано 5 закрытых отверстий) в одном из которых, возможно, лежит пачка денег. Тебе завязывают глаза, крутят барабан и после остановки ты можешь наугад открыть любые 2 (или может 3) ящика. Если в этих ящиках нет денег, то ящики закрывают и процесс повторяют. То есть каждый раз после остановки барабана ты не знаешь какие именно ящики уже проверял раньше.

Задача в том, чтобы найти схему открывания ящиков, которая гарантирует, что ты проверишь все ящики за конечное число попыток. Помню, там было решение и табличка со схемой по типу открываем 2 соседних ящика, в следующий раз через один и т.п. В итоге через 4-5 попыток вроде как получалось, что проверялись все ящики, независимо от начальных положений. Сейчас пробовал воспроизвести решение и нифига не вышло. Получается всегда есть шанс, что барабан будет останавливаться таким образом, что ты всегда будешь проверять одни и те же ящики.

Кто-нибудь видел такую задачу? Или может знает, как ее решить?
Аноним 26/05/24 Вск 15:06:16 115182 278
>>115181
Сформулируй это как игру с нулевой суммой, где один игрок ставит шаблоны, а оппонент выставляет призы в начале и "крутит" барабан.
Аноним 26/05/24 Вск 15:58:47 115183 279
>>115169
>трансфере
обмене
>комплексных
сложных
>менторства
наставничества
>легальной
законной
>аффилировано
сведено
Аноним 26/05/24 Вск 16:01:20 115184 280
>>115183
Три из пяти неправильно. Ты захотел выебнуться, а в итоге обосрался.
Аноним 26/05/24 Вск 16:06:16 115185 281
>>115184
>Три из пяти неправильно.
Уточняй.
>Ты захотел выебнуться, а в итоге обосрался.
Зачем мне выёбываться при понторезе? Достаточно лишь указать что понторез сам выёбывается и уточнить в чём, что успешно получилось.
Аноним 26/05/24 Вск 16:31:08 115186 282
>>115182
Сформулирую это так. Допустим тебе разрешили открывать 4 из 5 ящиков в каждой попытке. То есть, чтобы гарантированно обыскать все ящики тебе просто нужно в начале второй попытки знать, какой из ящиков ты еще не открывал в первой. Иначе есть вероятность, что во второй попытке ты откроешь те же самые 4 ящика, что и в первой. Твои действия?
Аноним 26/05/24 Вск 18:30:24 115187 283
>>115185
Высер другого анона я даже не читал, всё беру без контекста.
Трансфер в общем случае односторонний/в одном направлении, обмен двусторонний.
"Сложный" может иметь другие сопутствующие смыслы, нередко лишние. Комплексный может означать напр. состоящий из частей, характеристика которая может быть ортогональна "сложности" системы/процесса.
Аффилировано/сведено тут ты вообще проебался, видимо даже на ангельском значение не вкурил я полагаю. Affiliated это может быть ассоциируется с или связано с.

Другой анон я думаю сознательно троллит и засирает доску. Таких конечно репортить. Но лично я долбоёбов, которые надменно агрятся на заимствования (особенно невежественных, как ты), терпеть не могу ещё больше.
Аноним 26/05/24 Вск 20:16:21 115188 284
>>115187
>всё беру без контекста.
>Комплексный может означать напр. состоящий из частей
Тогда завали ебало, раз не выкупаешь контекст. Дальше читать не стал.
Аноним 26/05/24 Вск 20:51:02 115189 285
>>115188
хуя у тебя тряска
слив засчитан
Аноним 26/05/24 Вск 21:24:31 115190 286
>>115181
Если это возможно, то больше четырёх попыток тут явно не нужно, у тебя тут всё по модулю 5.
У меня вышло вроде бы вручную это методом исключения посчитать, начинал рассуждения как-то так: первые ящики (назову их реперными, относительно которых сдвигом открываем вторые) все 4 попытки совпадать не могут, так как иначе сдвиги весь круг покроют. Так что точно есть два разных. Тогда можно показать, что как минимум один из сдвигов будет не совпадать с этими двумя рэперными ящиками. Дальше примерно из тех же соображений можно тогда показать, что есть ещё как минимум один реперный ящик, который отличается от первых двух. Ну и вроде можно это до конца довести.
Но это, даже если верно, довольно некрасивое решение. Можно попытаться по индукции доказать, но нужно посмотреть, играет ли тут роль простота числа 5 (вроде нет). Или из более элегантных соображений, по противному мб.
Аноним 26/05/24 Вск 21:30:38 115191 287
>>115188
Другой анон просто тупой тролль, а ты реально ебанько.
Аноним 26/05/24 Вск 21:33:50 115192 288
>>115189
>>115191
Мелкобуква притворяется двумя долбаёбами, которые не выкупают контекст. Спешите видеть.
Аноним 26/05/24 Вск 21:51:22 115193 289
>>115190
>первые ящики (назову их реперными, относительно которых сдвигом открываем вторые)
Ну так основная проблема в том, что после повторного вращения ты уже не знаешь какой ящик у тебя был реперным. Я даже в этой >>115186 ситуации не знаю как можно действовать.
Аноним 26/05/24 Вск 22:32:01 115194 290
>>115193
Реперными я называю просто те, которые выбираю первыми, на $i$-ом шагу, относительно них делаю сдвиг в одну и ту же сторону на $i$. В итоге получается набор $\{x_i, x_i+i | i=1,...,4\}$. Реперными я называю те, которые $x_i$. Они могут быть априори какими угодно, но можно показать, что одинаковыми они точно не могут быть и т.д.
Аноним 27/05/24 Пнд 06:06:12 115195 291
Что-то не понимаю. Почему интервал (0,1) счетный?
Аноним 27/05/24 Пнд 07:22:31 115196 292
>>115195
Какими элементами представлен интервал?
Аноним 27/05/24 Пнд 07:44:57 115197 293
17167180754200.jpg 13Кб, 170x162
170x162
>>115194
Я понял что ты называешь реперными точками. Я не понял в чем идея сдвигаться относительно реперной точки, если ты не знаешь какая точка было реперной на шаге i-1.

Допустим пронумеровали ящики с внутренней стороны крышки. Ты не видишь эти номера когда выбираешь какие ящики открывать. Рассмотри ситуацию, когда можно открывать сразу 4 ящика за попытку. Каким образом ты планируешь гарантированно открыть 5-й? Для любых комбинаций открываемых ящиков всегда можно выбрать такие реперные точки, что 5-й ящик так и останется закрытым, а значит ты не можешь гарантировать его открытие за конечное число шагов, получается только вероятность.

Допускаю, что я упустил какую-то часть условия. Поэтому и задал тут вопрос, что может кто-то встречал такую задачу. Но точно помню, что ещё тогда она показалась мне контринтуитивной и я с удивлением обнаружил, что она всё-таки имеет решение.











Аноним 27/05/24 Пнд 08:58:17 115198 294
Ахаха. Стоило только написать, что возможно я не совсем точно помню условие, и я его вспомнил. Лол. Можно открывать только 1 ящик, после каждой попытки барабан с ящиками смещается случайным образом, но только на 1 позицию. Либо влево, либо вправо.

Имаджинирую, кстати, сколько еще ахуительных советов понаписывали тут "математики", которые поясняли мне решение не решаемой задачи и игнорили мой контрпример.
Аноним 27/05/24 Пнд 10:50:02 115199 295
>>115171
Все великие ебали тяночек. Галуа тоже за зрелую тяночку с молочными сиськами и отросшей жопой умер, а не за малолетку и тем более шотика.
Из притона где Шольце воспитывалися тоже ни одной подобной новости.
Это всё московские извращенцы-недочеловеки. Меня выворачивает наизнанку, когда слышу "московская математическая школа". Сразу веет чем-то совковым, грязным, насильственным, гомоеблей в сраной палатке и вазилином в качестве смазки.
Лучше тяночек ничего нет.
Аноним 27/05/24 Пнд 11:07:14 115200 296
>>115198
В математике про большинство задач неизвестно, решаемы они или нет.
Аноним 27/05/24 Пнд 11:12:51 115201 297
>>115198
Алсо первый же анон по сути и ответил, что для любой конфигурации открытия можно подобрать поворот так, чтобы приз не был открыт.
Аноним 27/05/24 Пнд 11:40:51 115202 298
>>115201
Про поворот это я сам и написал. Я же говорю тут >>115198 про свой контрпример. А первый гений >>115182 мне написал какую-то бессмысленную залупу, зато с отсылкой к теории игр.

Короче, вопрос закрыт. Не буду больше вас отвлекать от важного обсуждения ебли в жопу среди математиков.
Аноним 27/05/24 Пнд 11:59:56 115203 299
>>115202
Про нулевую сумму я тоже не понял, но остальная идея у него верная.
Аноним 27/05/24 Пнд 12:03:33 115204 300
>>115202
Так это доска по математике. Пучки, многообразия, схемы. Ты бы ещё в /pa спросил. Тебе на доску любителей заковырных задачек.
Аноним 27/05/24 Пнд 12:46:01 115205 301
>>115197
Ну да, я по сути просто показал, что можно открыть всегда три разных ящика из очевидных соображений, что если взять два соседних и два не соседних, то это будут три разных ящика как минимум, подумал, что можно и дальше так продолжать. Но стальные конфигурации так не отличить из-за симметрий.
Аноним 27/05/24 Пнд 13:59:22 115206 302
Мораль, старая как доска: если какой-то таракан задаёт вопрос про вероятности, ящики или какие-то ещё говно в этом роде, гнать его в /pr, или сразу нахуй (по вкусу), или игнорировать тупо (я обычно так делаю)
Аноним 27/05/24 Пнд 14:26:55 115207 303
>>115206
>вероятности
>гнать его в /pr
Ничего не смущает?
>игнорировать
>(я обычно так делаю)
...
Аноним 27/05/24 Пнд 14:35:10 115208 304
Аноним 27/05/24 Пнд 14:51:25 115209 305
>>115207
Вероятности, Монте-Карло и прочая статистика это CS, они там это дрочат, от прогеров графики до нейросеточников. Так что лучше спросить в /pr.
Аноним 27/05/24 Пнд 15:46:10 115210 306
Забавно, в двух последних филдсах было по одному человеку, которые занимались теорией вероятностей.
Аноним 27/05/24 Пнд 19:37:13 115211 307
>>115184
он захотел потроллить и своего добился
Аноним 27/05/24 Пнд 21:02:03 115212 308
Есть сундук, если его открыть, то с вероятностью
0.5 там не будет ничего
0.3 - слиток золота
0.2 - два сундука с такими же шансами что-то достать
Какова вероятность получить из одного сундука (открывая вложенные сундуки) ровно n золотых слитков?

Для n = 1:
Пусть вероятность x, тогда x = 0.3 + 0.2(1 - (1-x)^2 - x^2)
x≈0.395644
Но если тупо написать программу и просимулировать миллион открытий сундуков, то у меня выдает вероятность ~0.387, что не похоже на то что я получил аналитически.

Вопросы:
Где я наебался с n = 1?
Как решить в общем случае?
Аноним 27/05/24 Пнд 22:54:11 115213 309
На сколько хорошая идея погуглить конференции на которые есть доступ людям из вне (пусть и за плату), чтобы там с кем то познакомиться? Я был на конференции по проге и там после докладов был social brake и все говорили вообще на разные темы, не только академические. А после вообще в бар кто то поехал. Но там и многие знакомы друг с другом давно. Как у маетматиков это происходит?
Аноним 27/05/24 Пнд 23:21:47 115214 310
>>115213
знакомиться на конференциях - важно и правильно
Аноним 27/05/24 Пнд 23:34:02 115215 311
>>115214
я имею ввиду знакомиться не для поиска научрука или коллеги, а просто пообщаться. Норм?
Аноним 27/05/24 Пнд 23:40:47 115216 312
>>115215
да, конечно, так и нужно
с ровесниками особенно
Аноним 28/05/24 Втр 00:04:52 115217 313
>>115216
Понял. Круто. Спасибо
Аноним 28/05/24 Втр 00:05:24 115218 314
Gnedenko BV.jpg 271Кб, 765x1024
765x1024
>>115209
>Вероятности, Монте-Карло и прочая статистика это CS
Да у тебя же помутнение разума, болезный.
Аноним 28/05/24 Втр 00:07:42 115219 315
>>115211
нет. ты не прав, дурачок.
Аноним 28/05/24 Втр 00:56:02 115220 316
Снимок экрана 2[...].png 182Кб, 988x372
988x372
Не лепо ли бяшеть братие
почяти старыми словесы
помогать тупорылым школьникам
метящим в математики?
(с)

Я не могу решить пункт 3) пикрила. Это "алгебра" Гельфанда.
Положим, что a/b < b/c (соответственно ac - bd = -1)
Было такое наитие: предположить, что такая дробь существует. Из результатов пункта 2) следует, что также между дробями a/b и b/c обязательно существует также соседняя a+b/b+c.
Дальше была мысль исследовать отношение между e/f и a+b/b+c.
e/f точно не может быть равна a+b/b+c, ведь тогда f=b+c, а по условию f<b+d.
Пусть e/f < a+b/b+c. Далее вычтем a/b: e/f - a/b < a+b/b+c - a/b. Из этого соотношения (учитывая ac - bd = -1) получаем, что (eb-af)(b+d)<f. Мы близки к противоречию! Только с (eb-af) надо что-то сделать. На этом я застопорился.
Аноним 28/05/24 Втр 00:58:39 115221 317
Как я вообще понял, в пункте 3) по сути просят доказать, что между a/b и c/d нет дробей, чей знаменатель был бы меньше b+d.
Аноним 28/05/24 Втр 01:51:24 115222 318
>>115220
едва ли возможно доказать, что число не находится между двумя заданными числами, только лишь сравнивая его с третьим числом (которое между ними находится)

ты можешь получить, вероятно, что твоё число меньше или больше этого третьего, но отсюда не будет следовать, что оно выходит за границы интервала
Аноним 28/05/24 Втр 06:03:25 115223 319
>>115220
>получаем, что (eb-af)(b+d)<f. Мы близки к противоречию! Только с (eb-af) надо что-то сделать.
это последнее число положительно по предположению о [math]a/b < e/f[/math], а значит, тем более получаете искомое [math]b + d < f[/math].
>>115221
а я вот что-то так и не смог понять, к чему эти задачи и зачем нужны соседние числа.
>>115222
в его задаче много дополнительных условий
Аноним 28/05/24 Втр 11:25:38 115224 320
>>115218
Во всех западных универов статистику и пр вероятности изучают на кафедрах статистики, а не на кафедрах математики.
Аноним 28/05/24 Втр 13:37:52 115225 321
>>115223
>я не смог понять
>зачем нужны соседние числа.
речь идёт по сути о специальной группе матриц размерности 2 с целым коэффициентами. это вполне интересно само по себе. кроме того, у задачи есть содержательная геометрическая интерпретация, не знаю, это ты сможешь понять или нет

>в его задаче много дополнительных условий
они никак не помогут: опровергнуть $a < x < b$, только лишь сравнивая $x$ с какими-то третьим $c$, не получится
Аноним 28/05/24 Втр 13:55:27 115226 322
>>115225
>только лишь сравнивая $x$ с какими-то третьим $c$, для которого $a < c < b$
Аноним 28/05/24 Втр 14:06:16 115227 323
>>115212
Уже сам всё понял.

Короче пусть шанс достать что-то x, его нетрудно найти из

x = 0.3 + 0.2(1 - (1-x)^2)

f это искомая функция

f(0) = 1 -x

f(1) = 0.3 + 0.2∗2∗f(0)∗f(1)

Из этого несложно вывести f(1)

f(n)=0.3 + 0.2(1 - prod(i=0; n; 1 - f(n-i)f(i)))

хз как написать произведение но вы поняли

f(n)=0.3 + 0.2(1 - (1 - f(n)f(0))^2 * prod(i=1; n-1; 1 - f(n-i)f(i)))

Ну это квадратное уравнение, которое тоже решается. Мне лень считать что там по итогу получится, но я решил.
Аноним 28/05/24 Втр 17:01:36 115228 324
Gnedenko BV.jpg 271Кб, 765x1024
765x1024
Аноним 28/05/24 Втр 20:29:18 115229 325
Снимок экрана о[...].png 74Кб, 795x388
795x388
Снимок экрана о[...].png 13Кб, 821x101
821x101
Сап. Решал следующую задачу по вариационному исчислению: $I[y]= \int_{0}^{1} [y'^2 - y(y')^3]dx.$ А(0;0), В(1;0) Необходимо было проверить, включается ли экстремаль в поле для экстремалей. В процессе решения уравнения Эйлера-Лагранжа пришел к одному из решений у'=0, а оставшуюся часть не решал в виду отсутствия необходимости + диффур довольно сложный был. Суть в том, что дальше я проверил на включение в поле через условие Лежандра из задачника Краснова (пик 1) и привел пример такого поля y=Const. Препод заявил, что условие написано в сборнике не до конца верно. Контраргумент от него: $ \int_0^2 (y^2+y'^2)dx, $ y(0)=0, y(2)=0. Не уверен, что делаю все верно, но по условию Якоби он также включается ($u=C(e^x-e^{-x})$, однако в том же Краснове есть точный контрпример (пик 2). Для него условие включения в поле экстремалей Якоби примет вид $u=C sinx$ и u обратится в нуль в pi. Помогите разобраться, что я делаю не так с этими условиями или в чем ошибки в задачнике. Гюнтера, Эсгольца, Ахиезера, Фомина пробовал разбирать, там либо не затронута эта тема, либо условие такое же, но, вероятно, подтягивается что-то с прошлых тем, которые я просто не успею все разобрать
Аноним 28/05/24 Втр 21:00:31 115230 326
>>115228
ну он прав отчасти
я в вильнюсе учусь, у нас отдельно кагбе pure mathematic и отдельно статистика, вероятность и актуарная математика
Аноним 28/05/24 Втр 23:04:33 115231 327
>>115230
Посылаю тебя на квантор всеобщности.
Аноним 29/05/24 Срд 00:56:02 115232 328
Продолжу. Где искать конференции на которые можно прийти послушать?
Аноним 29/05/24 Срд 12:36:10 115233 329
1665264943228.png 715Кб, 1039x734
1039x734
Аноним 29/05/24 Срд 19:11:09 115234 330
ASkGirlMod.jpg 73Кб, 524x388
524x388
>>29047 (OP)
Сап, недавно видел статью в которой доказывалось, что любое отображение из R -> R непрерывно если оно открыто и замкнуто одновременно. А теперь её найти не могу, может кто знает.
Аноним 29/05/24 Срд 19:25:10 115235 331
>>115234
> любое отображение из R -> R непрерывно если оно открыто и замкнуто одновременно
Хз что имеется в виду под открытым/замкнутым отображением, но среди множеств одновременно открыты и замкнуты только $\mathbb{R}$ и $\varnothing$, и на R твоя хуйня ломается: отображение f(x) = sign(x) - x, f : R -> R не непрерывно
Аноним 29/05/24 Срд 20:14:37 115236 332
>>115235
открытое/замкнутое отображение — это то, которое переводит открытые/замкнутые множества в открытые/замкнутые множества соотвественно

про утверждение>>115234 я не слышал, сказать сразу мне трудно
Аноним 29/05/24 Срд 21:23:24 115237 333
>>115235
>но среди множеств одновременно открыты и замкнуты только [mаth]\mathbb{R}[/mаth] и ∅
От топологии зависит, но в той статье вроде стандартная топология была на действительных числах, где базовое множество это открытый шар радиуса r.
>>115236
Там как то обыгрывалось, то что чтобы сделать из открытого отрезка [mаth](a,b)[/mаth] замкнутый, надо туда всего две точки добавить a и b.
Аноним 30/05/24 Чтв 18:13:08 115238 334
Аноним 30/05/24 Чтв 20:48:19 115239 335
Существуют ли скалярные произведения над $\mathbb{R}^n$, кроме поэлементного умножения координат: $x \cdot y = x^T y$ ? Можете привести примеров?
Аноним 30/05/24 Чтв 21:07:33 115240 336
>>115239
с точностью до изоморфизма (замены базиса) оно единственно
Аноним 30/05/24 Чтв 21:15:52 115241 337
>>115240
Во первых - есть ли пруфы?
А во вторых - зачем тогда его определяют через кучу аксиом, если оно все равно одно единственное? Не проще ли было бы тогда определить его через это покоординатное произведение?
Аноним 30/05/24 Чтв 21:23:37 115242 338
>>115241
ты не можешь доказать единственность какого-либо, если не дашь определения

>определить его через это покоординатное произведение
тогда мы привязаны к конкретным координатам, а что отвечает нашему объекту в общем случае, становится непонятно (поскольку в других координатах он меняется)
Аноним 30/05/24 Чтв 21:24:37 115243 339
>>115242
формула меняется, вернее, не объект
поэтому через формулу его не получается определить
Аноним 30/05/24 Чтв 21:27:26 115244 340
>>115243
и я наверно тебя путаю

более точно так: на заданном векторном пространстве скалярных определений можно определить много. но любое из них приводится к стандартному подходящей заменой координат
Аноним 30/05/24 Чтв 21:59:17 115245 341
image.png 59Кб, 1115x427
1115x427
>>115244
Я правильно понял, что для вообще любого скалярного произведения существует такая матрица $A$, что $x \cdot y = (Ax)^TAy$?
Аноним 30/05/24 Чтв 22:00:03 115246 342
>>115245
ой, пирикл от другого поста
Аноним 30/05/24 Чтв 22:07:40 115247 343
>>115242
определение через аксиомы:
$x \cdot y = \overline{y \cdot x}$
$\alpha(x \cdot y) = (\alpha x) \cdot y$
$x \cdot (y + z) = x \cdot y + x \cdot z$
$x=0 \Rightarrow x \cdot x = 0 \wedge x \neq 0 \Rightarrow x \cdot x > 0$
Аноним 30/05/24 Чтв 22:14:22 115248 344
Павда ли, что для любых матриц А и В: $\overline{A \cdot B} = \overline{A} \cdot \overline{B}$? Можете доказать?
Аноним 30/05/24 Чтв 23:27:29 115249 345
Аноним 30/05/24 Чтв 23:38:19 115250 346
2024-05-30-2329[...].png 39Кб, 600x130
600x130
можете мне объяснить что здесь написано? книга: техническая механика, 2012, ладогубец, лузик.

судя по "вопросам для самоконтроля", в первом имелось ввиду что деформируемое тело в равновесии будет в равновесии если бы было и не деформируемое.

но что во втором я не могу расшифровать. что-то типа если приложить к связи (препятствию) силу ограничивающую тело (реакция связи), то оно станет свободным?
Аноним 31/05/24 Птн 10:07:02 115253 347
image.png 170Кб, 1024x767
1024x767
кто то писал, что не бывает уравнений отрезков. нашол вот такую картинку. Но на десмосе график всё равно не построился
Аноним 31/05/24 Птн 10:58:29 115255 348
>>115239
Смотри. У тебя есть то что ты мыслишь как координатная плоскость и радиус-векторы. А есть координаты пары чисел $(x,y)$. Они вроде как связаны, но вообще разные объекты. Пары чисел это пары чисел. Ты можешь за основу взять не стандартный базис с перпендикулярными осями, а косоугольный какой-нибудь, тогда у векторов будут другие координаты, но сами вектора как бы остаются теми же, их длина и углы меж ними не меняются.
$(x,y)$ это просто пары чисел и ты можешь их складывать и на число умножать. У этих пар нет никакой геометрии, как у векторов на плоскости. Нет длины, нет углов. Это просто числа-буквы.
Хорошо бы перенести эти понятия с геометрического образа на эти пары чисел, чтобы работать с символами.
Ты заявляешь, что "длина", точнее её квадрат, $(x,y)$ равняется $x^2+y^2$, перенося теорему Пифагора на символы.
Аналогично ты вводишь понятие "угла" меж векторами, для этого придется воспользоваться косинусом. Если нужно могу в деталях расписать как это делается. Получишь стандартную формулу через координаты и в тоже время бескоординатную форму. Получишь скалярное произведение.
Дальше ты можешь заметить, что ты можешь длину вектора получить с помощью скалярного произведения. Тогда скалярного произведения достаточно, чтобы вычислять углы меж векторами и их длины. Оно как бы наделяет пространство пар-чисел геометрией и можно за основу взять не длину с углами а скалярное произведение.
У произвольного векторного пространства нет координат. Это просто буквы: $u,v,w...$ которые можно складывать и домножать на числа. Если бы у нас на нем было задано скалярное произведение, то это пространство заимело бы геометрические свойства. Появились бы длины векторов и углы меж ними.
Для этого скалярное произведение просто обобщили. Посмотрели, что скалярное произведение это билинейная положительно определенная форма. И в таком виде скалярное произведение можно приложить уже к любому пространству, главное чтобы аксиомы выполнялись. Естественно этих форм дохуя и больше. Но всегда можно, например, для любого пространства с формой $(V,g)$ найти ортогональный базис. И в этом базисе скалярное произведение будет записано стандартной формулой через координаты. Впринципе и в других координатах будет та же формула только коэффициенты появятся.
На пространстве функций скалярное произведение можно определить через интегралл.
Аноним 31/05/24 Птн 11:57:07 115256 349
>>115255
О, вроде все понятно
Аноним 31/05/24 Птн 13:54:33 115257 350
>>115256
Про углы.
Есть плоскость $R^2$ с стандартной школьной системой координат.
Длину по координатам легко определить
$|v|^2=x^2+y^2$
С углами сложнее. Напрямую мы угол с координатами связать не можем. Но может связать посредством триг. функций. Пусть $v$-вектор и $ф$ угол между $v$ и осью $OX$, тогда
$cos(ф)=\frac{x}{|v|}$ $ф$ обычно называют аргументом $v$. Аналогично
$sin(ф)=\frac{y}{|v|}$
Почему берем косинус а не другие функции будет ясно дальше.
Нам важно находить угол между векторами. Тк мы не можем работать напрямую с углами, то работаем с триг. функциями.
Пусть $u,v$ два вектора и между ними угол $w$. Пусть $т$ аргумент $u$ и $ф$ аргумент $v$, тогда
$w=ф-т$
Заметь что эта величина может быть как положительной, так и отрицательной. Тогда
$cos(w)=cos(ф-т)=cos(ф)cos(т)+sin(ф)sin(т)$
Подставляем вместо косинусов углов справа их выражения через координаты.
$cos(w)=cos(ф)=(\frac{x_{v}}{|v|})(\frac{x_{u}}{|u|})+(\frac{y_{v}}{|v|})(sin(ф)=\frac{y_{u}}{|u|})
$|u||v|cos(w)=x_{u}x_{v}+y_{u}y_{v}$
Мы берем косинус, потому что $cos(a)=cos(-a)$, в то время как $sin(-a)=-sin(a)$. В случае если бы мы выбрали $sin$ у нас скалярное произведение зависело бы от порядка множителей тк $w=ф-т$, поменяя порядок получим $w=т-ф=-(ф-т)$. На самом деле понятно, что это не мешает определить угол между векторами. Но косинус просто приятнее в силу что мы всегда получаем одно и тоже число не смотря на порядок множетелй. Так же косинус позволяет вычислять длины, тк $cos(0)=1$, в то время как $sin(0)=0$, тогда $|v|^2=v.v$ и мы можем отказаться от углов и длин и за основу взять произведение $u.v$ и выводить эти вещи из него.
Легко проверить что на плоскости скалярное произведение билинейно $u.a(v+v')=a(u.v+u.v')$ и положительно определенно, то есть $v.v\ge0$, если вектор не нулевой.
Эти свойства берутся и обобщаюстя - скалярное произведение просто билинейная положительно определенная форма $B(u,v)$ дйствующая на веткорном пространстве.
Аноним 31/05/24 Птн 14:53:38 115258 351
>>115253
возьми $(x,y) = (3a/4, -a/4)$ - эта точка лежит на нарисованном отрезке; для простоты можешь взять $a = 4$
Аноним 31/05/24 Птн 19:06:38 115259 352
image.png 192Кб, 630x591
630x591
Какого хуя число называют полуосью? Математики там совсем укуренные?
Аноним 31/05/24 Птн 19:11:37 115260 353
>>115259
непонятно, почему ты решил, что этот текст писали математики
Аноним 31/05/24 Птн 19:21:35 115261 354
>>115260
Это общематематический термин, причем тут этот текст?
Аноним 31/05/24 Птн 19:32:03 115262 355
>>115261
ты глупость какую-то написал, даже определитель матрицы одни авторы поясняют как огромную формулу с грудой значков, другие - как специальное отображение, заданное набором свойств, третьи - как значение функтора высшей внешней степени на операторе, а четвёртые - как естественное преобразование между специальными функторами
Аноним 31/05/24 Птн 19:45:08 115263 356
>>115262
Это все эквивалентные определения одного и того же объекта
Аноним 31/05/24 Птн 19:49:23 115264 357
>>115263
ну ты можешь и число в эллипсе называть любыми словами, и это всё будут эквивалентные определения, что не так?
Аноним 31/05/24 Птн 20:06:32 115265 358
>>115264
То есть тебе норм, что число называют осью?
Аноним 31/05/24 Птн 20:48:07 115266 359
>>115265
это претензия к авторам твоего текста
Аноним 31/05/24 Птн 20:54:34 115267 360
>>115266
А что такое большая полуось по-твоему?
Аноним 31/05/24 Птн 21:05:02 115268 361
>>115267
не знаю, мне это неинтересно
Аноним 01/06/24 Суб 15:52:11 115269 362
Screenshot2024-[...].jpg 251Кб, 1080x1465
1080x1465
Двач, что делать, если тупой?

Хочу поступить в магистратуру по аналитике данных:

https://karpov.courses/big-data-analytics

Но там скинули демо вступительных, пиздец полный. Несмотря на то, что есть опыт работы в этом направлении, матан пизда. За сколько можно к такому подготовиться? На пике скрин с демо
Аноним 01/06/24 Суб 16:51:33 115270 363
>>115269
это элементарные задания, времени слишком много научиться их делать занять не должно
Аноним 01/06/24 Суб 16:57:14 115271 364
>>115270
А не мог бы посоветовать, как подготовиться, с какими материалами?
Аноним 01/06/24 Суб 17:46:53 115272 365
>>115271
можешь попробовать почитать что-нибудь вроде "математика для инженеров"
Аноним 01/06/24 Суб 18:11:31 115273 366
Верно ли будет сказать, что групповое сопряжение - это то, как преобразуется левое умножение в результате (другого) левого умножения? Можно это как-то категорно описать?
Аноним 01/06/24 Суб 18:12:35 115274 367
>>115273
Под категорно я имею в виду, что может понятие есть какое. Коммутативную диаграммку нарисовать не проблема.
Аноним 01/06/24 Суб 18:33:17 115275 368
>>115269
за 11 классов можно, а лучше 11 классов и 2 курса математической специальности бакалавриат
Аноним 01/06/24 Суб 19:35:10 115276 369
>>115269
Лол. Там у них написсано, что проходят матан и линал в 1м семе. Почему это магой называется - хз. Мб это для гуманитариев? Но скорее развод гоев
Аноним 01/06/24 Суб 20:20:42 115277 370
>>115269
Не, 10-классник справится. Прочти как брать производные и как исследовать с помощью них функции. Ничего сложного тут нет.
Аноним 01/06/24 Суб 20:28:14 115278 371
>>115273
Я не понимаю что ты имеешь ввиду. Ты хочешь понять "смысл" сопряжения?
Аноним 01/06/24 Суб 22:03:03 115279 372
>>115278
Да. Потому что везде написано одно и то же, и мне этого не было достаточно для интуитивного понимания. Сопряжение это по сути как подобие матриц при смене базиса. То есть мы "меняем базис", ну или переименовываем объекты - я это в терминах действия для себя представляю. И это индуцирует изменение вида левого умножения.
Я просто очень удивлён, что ничего толкового найти нельзя. Перерыл уже кучу книг. Видел что-то похожее в паре книг для физиков, и в стэкиксчендж постах.

Вот и спрашиваю, может это частный случай какой-то более общей идеи.
Аноним 01/06/24 Суб 22:09:35 115280 373
Привет, анон. Я в матёше вообще ни бум-бум, но праздное любопытство возникло, на фоне просмотра видосов по школьной геометрии.

Вот есть, допустим, на плоскости координат правильный многоугольник вписанный в окружность с радиусом, ну пусть 1/2(x).
Как математики выразили бы приближение стороны многоугольника к окружности?
А к точке начала координат? Как отразили бы случаи, когда многоугольник ближайшей точкой к 0 имеет точку на стороне, а когда смотрит к 0 углом? Точка угла же, по идее, должна же быть ближе точки на стороне? Наверняка же для этого случая что-то придумали, но нагуглить не вышло.

Сорямба, если глупости говорю, просто представляю картинку, и интересно стало.
Аноним 01/06/24 Суб 22:11:55 115281 374
С увеличение количества сторон, имеется в виду.>>115280
Аноним 02/06/24 Вск 02:15:37 115282 375
>>115269
Ору. Циганок накидал задачек чтобы отсеять полных даунов. Ну и чтобы солидна было как в Херне. Но похоже все равно перестарался.

Вот чего я не понимаю почему додики из хуяндекса не обучают тупо всех желающих вместо того чтобы заставлять людей задрачивать невероятную четверть-финальную поебистику. Ресурсы то у них есть и наверняка все домашки проеверяются либо роботом либо на отъебись. Да раньше там одна насосавшая жидовка олимпиадница эту хуйную проталкивала, но теперь она вроде свалила в свой жидовник. Инертность порядков такая?
Аноним 02/06/24 Вск 05:57:49 115283 376
>>115279
Ну бывает чтобы понять что-то нужно погрузиться в историю.
Перестановки начал рассматривать Лагранж. Но их рассматривал не в вакууме, как делают сейчас, а в связи с функциями от $n$-аргументов, от корней уравнений.
Например $f(x,y)=x-y$ или $g(x,y)=x+y$. Мы считаем функции равными если может одну из другой получить с помощью законов алгебры: коммутативность сложения/умножения.
Для каждой функции можно записать ту группу перестановок, что оставляют её на месте. Например для $f$ из примера выше это ${e}$, единичная группа, а для $g$ это $S_{2}$. Перестановка $(1,2)$ меняет $f(x,y)$, превращает $f(x,y)$ в $-f(x,y)$ что в общем говоря другая функция.
Пусть теперь $f_{1}, f_{2}$ различные функции от $n$-аргументов, $f_{1}$ имеет группу $H$ и $f_{2}$ получается из $f_{1}$ какой-то перестановкой $g$ из $G$, для которой $H$ является подгруппой. Тогда естественна задача: какую группу имеет $f_{2}$?
Ответ на это прост: $gHg^{-1}$, попробуй доказать его сам.
1) $G$ является подгруппой $S_{n}$. Есть вопрос такой, есть ли в $S_{n}$ перестановки, не лежащие в $G$, которые переводят $f_{1}$ в $f_{2}$?
Зная группу $f_{1}$ и что $g(f_{1})=f_{2}$ найди все перестановки, так же переводящие $f_{1}$ в $f_{2}$
2) Пусть $H$ группа $f_{1}$ и $g(f_{1})=f_{2}$, найди группу $f_{2}$
Аноним 02/06/24 Вск 09:23:45 115284 377
>>115283
Спасибо анон, другой взгляд всегда интересен. Когда пытался понять теорию Галуа, то тоже встречался с тем, что многие понятия появились из теории симметрических полиномов.

Твой пример это по сути следующее: у нас есть стабилизатор $f_1$, и мы знаем, что $f_2$ в одной орбите с $f_1$. Тогда стабилизатор $f_2$ это просто подгруппа, сопряженная стабилизатору $f_1$. Это я знаю. Но это просто ещё один пример, разве нет? Не вижу тут общности.
Но то, что любое действие это просто перестановка, это собственно и есть причина, по которой мне о сопряжении лучше думается в терминах смены базиса, или переименовывания элементов. Так что симметрические функции это отличный пример, но всё равно только пример.

Но я покурю ещё в сторону исторического развития, спасибо
Аноним 02/06/24 Вск 09:28:10 115285 378
>>115280
простая аналитическая геометрия, на университетском языке - ангем
координатная плоскость [math]\mathbb{E}^2[/math], на ней точки, точке соответствуют координаты парой [math](x, y)[/math] действительных чисел.
расстояние между точками [math]A (x_1, y_1), B (x_2, y_2)[/math] выражается значениями метрической функции [math]\rho[/math]. можно определить [math]\rho((x_1, y_1), (x_2, y_2)) = \sqrt{(x_1 - x_2)^2 + (y_1 - y_2)^2}[/math].
окружность с центром [math]O[/math] и радиусом [math]r[/math] представляет собой множество только любых таких точек [math]A[/math], что они удалены от [math]O[/math] на расстояние, равное [math]r[/math]: [math]\rho(A, O) = r[/math].
правильный многоугольник будет иметь периметр, состоящий из сторон, которые последовательно соединяют вершины [math]A_1, A_2, ..., A_n[/math], расположенные через равные расстояния [math]\rho(A_i, A_{i+1} = a[/math] на некоей окружности. при задании одной из вершин, координаты каждой из остальных вершин могут быть заданы как решения совокупности систем уравнений, выражающих принадлежность вершины описанной окружности и ее удаленность от предыдущей вершины на расстояние, равное длине стороны многоугольника. каждая сторона многоугольника представляет собой только любые такие точки, что сумма расстояний от точки до соответствующих вершин равна длине сторогы.
думаю, из этого уже должно быть понятно, как выразить понятия приближения при изменении числа сторон.
Аноним 02/06/24 Вск 09:39:49 115286 379
>>115284
Ты думаешь не так. Есть 2 способа решать задачи.
1) Ты изучил тему Х, видишь задачу х, и думаешь как применить Х к х.
2) Ты задачу х пытаешься рассмотреть как бы с нуля, и пытаясь её решить приходишь к Х и применяешь.
>Но то, что любое действие это просто перестановка, это собственно и есть причина, по которой мне о сопряжении лучше думается в терминах смены базиса, или переименовывания элементов.
Такая точка зрения есть в Алексееве-Теорема Абеля.

>Так что симметрические функции это отличный пример, но всё равно только пример.
Так это пример откуда и выросли группы. Представь что сейчас 19 век. Смысл рассматривать пересатновки? Их стали применять только когда они показали эффективность в теории Лагранжа.

Всю теорию групп придумал Лагранж с Галуа, и все определения они выводили именно из примеров симметрических многочленов. Лагранж в общем-то почти в соло доказал теорему Абеля, но он слишком к функциям привязался. А Галуа заметил, что вместе с "расширениями" функций их группа падает.
Например решая квадратные уравнения мы можем вычислять только симметричные функции.
Мы берем $f(x,y)=(x-y)^2$, её можно выразить через элементарии, и затем, взяв корень, мы получим в арсенал не только симметрические функции, но и класс пошире. Но взамен группа падает. Это лучше видно на примере решения куб. уравнения, тк мы расширяем поле в 2 шага, а не в 1.
Можно доказать, что если $f$ и $g$ имеют одну и ту же группу $G$, то $f$ рационально выражается через $g$ и симметрические функции.

Дальше эти конструкции просто перенесли на общие группы. Потому что если рассматривать перестановки сами по себе, нет никакого намёка на нормальные подгруппы, сопряжение, разрешимые. Эти все определения пришли из многочленов.
Аноним 02/06/24 Вск 09:50:31 115287 380
>>115286
>Такая точка зрения есть в Алексееве-Теорема Абеля.
Я знаю, это же Арнольд. У него это есть и в книге про гидродинамику, но слишком кратко.
>Дальше эти конструкции просто перенесли на общие группы. Потому что если рассматривать перестановки сами по себе, нет никакого намёка на
>нормальные подгруппы
>разрешимые
Это мне известно (не так хорошо, как тебе правда), да.
>сопряжение
Для меня есть большая разница между понятиями нормальной подгруппы, разрешимой, и сопряжения. Без первых двух можно жить, когда речь идёт о действиях, а без сопряжения никак. Например меня интересуют многообразия и группы Ли: сопряжение постоянно используется для представления алгебры Ли. Коммутатор - линейное приближение сопряжения. Не вижу, как симметрические полиномы мне помогают понять, почему сопряженное представление алгебры Ли имеет такую ключевую роль.

Но я ещё подумаю на досуге над твоим постом
Аноним 02/06/24 Вск 12:33:33 115288 381
Привет матач! Что бы почитать о статистике чтобы вкатится? Начал курсец на степике, но этого явно мало. Автор советует Гланца учебник по медицинской статистике. Этого хватит для вката?
Аноним 02/06/24 Вск 14:06:22 115289 382
>>29047 (OP)
Макарычев 7 класс, задача 108: при каких n из N 9n+1 кратно 11? Чёто не могу придумать нихуя
Аноним 02/06/24 Вск 14:45:27 115290 383
>>115289
Мне кажется или чатгпт несёт хуйню?

Остаток от деления −1 на 11 — это число, которое нужно добавить к −1, чтобы получить кратное 11.
...
Это означает, что остаток от деления −1 на 11 равен 10.
Аноним 02/06/24 Вск 15:20:16 115291 384
>>115289
Просто подставляешь в формулу (9n+1) числа от 1 до 10 (дающие всевозможные остатки при делении на 11) и получаешь что число 6 подходит. Значит подходящими являются любые числа вида 11k+6, где k - целое число.
Аноним 02/06/24 Вск 15:33:57 115292 385
>>115290
Да, он пиздит, остаток от деления - это такое число, которое нужно вычесть, чтобы получить кратное. Так -1 вычесть 10 будет -11, что кратно 11

Юзать гпт для математики - плохая идея, если кто-то еще не понял
Аноним 02/06/24 Вск 15:57:23 115293 386
>>115291
>>115292
Спасибо. Ну гпт тоже помог, дал верный ответ и расписал решение. Решению я не доверяю, но это хоть какая-то подсказка куда копать.
Аноним 02/06/24 Вск 16:04:41 115294 387
>>115292
>-11, что кратно 11

Кратность вроде определена только на натуральных числах.
Аноним 02/06/24 Вск 16:15:10 115295 388
Аноним 02/06/24 Вск 16:26:57 115296 389
Вырастает поколение дебилоидов, которое неиронично использует чатжпт как всезнающее око.
Аноним 02/06/24 Вск 16:28:36 115297 390
>>115294
Понятие делителя существует в произвольном кольце.
Аноним 02/06/24 Вск 16:29:21 115298 391
>>115296
Так если оно и есть всезнающее око. Даже со всеми косяками и ошибками.
Аноним 02/06/24 Вск 16:34:17 115299 392
image2024-06-02[...].png 64Кб, 729x737
729x737
Аноним 02/06/24 Вск 16:45:00 115300 393
>>115289
Альтернатива этому >>115291 анону
$9=11-2$
$(11-2)n+1=11n-2n+1$
Пусть число кратно, тогда $n-\frac{2n}{11}+\frac{1}{11}$ целое или $1-2n$ делится на $11$.
Все числа $11k$ кратны $11$, тогда $1-2n=11k$ и $n=-\frac{11k-1}{2}$
Аноним 02/06/24 Вск 17:22:24 115301 394
>>115300
Ну тогда уж сразу
9n + 1 = 11k
n = (11k - 1) / 9
И не дохуя пользы от такого решения, т.к. k зависит от n))
Аноним 02/06/24 Вск 17:47:16 115302 395
О, у жпт школотронов тряска.
Аноним 02/06/24 Вск 18:20:45 115303 396
>>115301
Ты дурак? Подставляешь $k$ и получаешь $n$
$k=1$ $n=-\frac{11-1}{2}=-5$ проверяем
$9(-5)+1=-44$
вау, делится на $11$
$k=-1$ $n=6$
$9\cdot6+1=55$
нихуя, опять делится на $11$
$k=3$ $n=-16$
$9(-12)+1=-143$ снова делится на $11$

Но правда решение анона лучше, тк у меня только нечетные $k$ подходят. Можно усовершенствовать, $k=2z+1$, тогда
$n=-\frac{11(2z+1)-1}{2}=-\frac{22z+10}{2}=-11z+5$
а если подставить $k=-2z-1$, то получим формулу анона
$n=-\frac{11(-2z-1)-1}{2}=-\frac{-22z-12}{2}=-(-11z-6)=11z+6$
Аноним 02/06/24 Вск 19:19:42 115304 397
Аноним 02/06/24 Вск 19:43:16 115305 398
>>115304
Что тебе не нравится? Я не понимаю. Я тебе дал формулу
$n=11z+6$ и как её получить.
Ставишь любое целое $z$ и получаешь $n$, которое подходит по условию.
У тебя ни $z$ ни $k$ выше не зависят от $n$, это просто параметры.
Аноним 02/06/24 Вск 23:44:47 115307 399
1662699763126.png 183Кб, 1071x796
1071x796
Пацаны, я решил серьёзно вкатиться в математику. Для начала хочу начать с азов: наверстать упущенное в своей мухосранской "гимназии" и максимально воспроизвести опыт обучения в матшколе: продвинутая теория по соответствующим учебникам, много практики, задач и доказательств - жёсткий дроч 24/7.
За образец беру 57-ю школу.
Заказал всё необходимое вроде.
Можете ещё что посоветовать?
Аноним 03/06/24 Пнд 06:07:24 115308 400
>>115307
Для начала отмени этот заказ (фаллоимитатор можно оставить) и потом подумай, что ты сделал не так, почему и как надо было сделать и почему. Это будет твоим первым упражнением.
Аноним 03/06/24 Пнд 08:40:15 115309 401
>>115307
пердиксовый юморок
Аноним 03/06/24 Пнд 11:57:17 115311 402
2445.jpg 10Кб, 236x236
236x236
Я все никак не пойму, что происходит с точки зрения логики, когда мы вводим новую переменную, решая например систему уравнений. И поэтому я не понимаю, как следить за равносильностью переходов.

Нужно нам, допустим, решить следующее (x^2 + y^2 = 4) & (x^3 + y^3 = 8) (1). Делаем стандартную замену u = x + y; v = xy; и... что дальше, логически?
После этого мы разве переходим к эквивалентной системе (x^2 + y^2 = 4) & (x^3 + y^3 = 8) & (u = x + y) & (v = xy) ? Да вроде нет, потому что её решение включает в себя 4 числа, а не 2.
Я не могу пока что смотреть на это НЕ как на следствие. А почему это следствие мне понятно, рассуждение следующее : если для некоторых x и y верно (1), то из этого следует, что существуют такие два числа u и v что u = x + y; v = xy. Ну и дальше мы работаем с этими нововведенными переменными. Но после уже следствия, то есть решения на x и y обязательно надо будет проверять!
Может есть какие-то книги, где логика происходящего, например в таких случаях, разжевывается? Сомневаюсь, что это стандартные учебники по мат. логике, потому что там какие-то дебри, а меня интересуют совершенно, вроде бы, базовые вопросы (ответы на которые всем, кроме видимо меня, настолько очевидны, что они по мнению решателей задач/писателей учебников даже не заслуживают упоминания).
Аноним 03/06/24 Пнд 12:04:58 115312 403
>>115311
равносильно, потому что если исходя из a = b заменили a на b, то можно и наоборот заменить b на a. следствие в обе стороны.
Аноним 03/06/24 Пнд 12:09:43 115313 404
Двач, я уже почти до тригонометрии дошёл, но есть один начальный момент который я недопонял.
Я так понимаю у функций одна форма записи для двух действий или я что-то не понимаю?
Функции (я так понимаю) присваивается значение: f(x) = a(x - h)^2 + k
И в этом-же правиле, предлагается решить: f(x) = 0
Здесь уже используется (подставляется) функция и устанавливается уравнение?
Аноним 03/06/24 Пнд 12:26:46 115314 405
>>115313
Нейросеть вроде подтверждает мои предположения, но хотелось бы услышать что люди скажут.

### Summary
Yes, the notation "f(x) =" specifically can be used for two distinct purposes:

1. Defining a function: f(x) = expression
2. Setting up an equation to solve: f(x) = value

Understanding the context in which "f(x) =" is used will help determine whether it is defining a function or setting up an equation to solve.
Аноним 03/06/24 Пнд 12:51:52 115315 406
>>115312
А тут он не просто а на бэ заменил, а x+y на u и xy на v, так что хуй его знает как оно там работает
Аноним 03/06/24 Пнд 13:05:27 115316 407
>>115313
ты изучаешь некую математическую теорию [math]T[/math], в которой есть сложение, вычитание и возведение в квадрат.
эта теория [math]T[/math] расширяется до теории [math]T_f[/math] добавлением
1) символа [math]f[/math] и обязательных логических аксиом с этим символом, таких как [math]x = y \to fx = fy[/math], [math]\neg (fx = 0) \vee (fx = 0)[/math], [math]f(5) = 20 \to \exists x (fx = 20)[/math] и мн. др. - их бесконечно много
3) и новой определяющей аксиомы - формулы:
>f(x) = a(x - h)^2 + k
благодаря этому в полученном расширении [math]T_f[/math] есть понятие о так определенной функции [math]f[/math]. вообще говоря, корректным является доказать существование и единственность [math]f[/math], что не всегда осуществляется так же тривиально, как в нашем случае.
затем в [math]T_f[/math] предлагается отыскать формулу [math]A[/math], и доказать в [math]T_f[/math] ее равносильность формуле
>f(x) = 0
: [math]\vdash_{T_f}[/math], причем, если используются обозначения теории множеств, то может требоваться, чтобы [math]A[/math] имела вид [math]x \in \emptyset[/math] или [math]x \in{a, b}[/math].
Аноним 03/06/24 Пнд 13:08:34 115317 408
>>115314
> хотелось бы услышать что люди скажут.
А зачем? Просто спроси ещё раз у чатжпт. Зачем вообще сюда приходить?
Аноним 03/06/24 Пнд 13:09:37 115318 409
>>115317
>Зачем вообще сюда приходить?
Ну ладно, больше не буду.
Аноним 03/06/24 Пнд 13:10:03 115319 410
>>115315
>
a и b я использовал как мета-переменные, вместо которых могут быть любые термы, хоть x+y, хоть xy, хоть [math]lim_{x \to \infty} 1/x[/math]
Аноним 03/06/24 Пнд 13:16:29 115320 411
>>115314
Ну в целом да, тут в первом говорят, что f - это такая функция, что $\forall x f(x)=...$
А во втором просят найти такое множество A, что $\forall x \in A: f(x)=0$
Аноним 03/06/24 Пнд 13:19:53 115321 412
>>115316
поправочка
затем в [math]T_f[/math] предлагается отыскать формулу [math]A[/math], и доказать в [math]T_f[/math] ее равносильность формуле
>f(x) = 0
: [math]\vdash_{T_f}A \leftrightarrow ( f(x) = 0 )[/math]
Аноним 03/06/24 Пнд 14:08:37 115322 413
>>115321
Ну, наверное стоит поблагодарить за попытку объяснить, написал ты не мало, но я вроде писал что "я почти до тригонометрии дошёл" которая изучается сильно раньше математической логики, и пишу я в треде для начинающих, т. е. предполагается что я очень слаб в математике, зачем ты мне всё это пишешь на непонятном мне языке я если честно не очень понял.
Аноним 03/06/24 Пнд 14:36:53 115323 414
>>115311
>Я все никак не пойму, что происходит с точки зрения логики, когда мы вводим новую переменную

Ничего не происходит. Это просто обозначения. Ты можешь не использовать их и решать как есть, просто неудобно.
Аноним 03/06/24 Пнд 14:41:40 115324 415
>>115313
Попробуй сформулировать свой вопрос, чтобы человек смог его понять. Скорее всего в процессе сам поймёшь ответ.
Аноним 03/06/24 Пнд 15:14:39 115325 416
>>115322
>зачем ты мне всё это пишешь на непонятном мне языке я если честно не очень понял.
Еслт язык непонятен, то просто вбей его в чатжпт, делов-то.
Аноним 03/06/24 Пнд 16:10:21 115326 417
>>115282
> одна насосавшая жидовка олимпиадница
Трахнул бы. Милая женщина, а в молодости вообще топ была
Аноним 03/06/24 Пнд 16:51:26 115327 418
>>115282
Неосилятор, спок.
Аноним 03/06/24 Пнд 18:16:52 115328 419
Я вот тут подумал, а насколько больше толку стране, цивилизации и науке было бы, если бы тех, кто достаточно хорош в математике, чтобы взять всерос или межнар, с 5 класса готовили не к олимпиадкам и решению уже решенных задач, а к науке...
Окей, таковых надо ещё выявить. Ну давайте тех, кто прошел на Эйлера классе в 7, например, будем задрачивать не ко всеросу, потому что нахуй это надо вообще - талант уже выявлен, а к ресерчу. И как результат в классе 11 получаем уже чуть ли не полноценного Ученого.
Аноним 03/06/24 Пнд 18:20:41 115329 420
>>115327
Надо быть совсем ебнутым на голову чтобы при наборе грузчиков требовать их пробежать марафон.
Аноним 03/06/24 Пнд 18:30:34 115330 421
>>115322
0. думаю, нам стоит попытаться разобраться в твоем вопросе.
у тебя есть формулы
> f(x) = a(x - h)^2 + k
>f(x) = 0
и ты считаешь, что их можно воспринимать двояко. чтобы преодолеть неопределенность, ты хочешь узнать, откуда появляется отличие.
я, отвечая на твой вопрос, говорю вот о чем.
1. есть формулы, они составляют язык.
если среди формул выбрать теоремы - это уже теория.
теоремы определяются после назначения аксиом. теоремами считаются только те формулы, которые являются аксиомами или логически выводимы из теорем (рекурсия).
исходя из нашей мотивации, ограничимся только этим и пойдем дальше.
(при желании ты можешь изучить это досконально. это занимает не больше 1 часа. я бы мог изложить все в одном посте. но сейчас идем дальше.)
2. вообще говоря, в твоем языке есть символы [math]-[/math] [math]+[/math] [math]^2[/math], но нету символа [math]f[/math]. этот символ ты добавляешь и получаешь расширение языка. также ты считаешь, что символ обозначает функцию, и поэтому удовлетворяет обычным логическим правилам (а на логико-математическом уровне это означает явное добавление бесконечного множества логических аксиом). так получается расширение теории, где уже есть функция [math]f[/math].
3. функция [math]f[/math] может быть определена неким образом. что такое определение? это введение новой аксиомы - определяющей аксиомы.
добавление аксиомы тоже расширяет теорию. определяющая аксиома может не быть логической, а нести в себе дополнительную информацию. как, например, свойства функции [math]\sin[/math], такие как периодичность и др., основаны на знании определения [math]sin[/math], а не являются простым логическим следствием, справедливым для любой функции.
таким образом, значение первой формулы
>f(x) = a(x - h)^2 + k
заключается в том, что по условию задачи она принимается как дополнительная аксиома к той алгебраической теории, которую ты изучаешь.
4. далее тебе дается формула
> f(x) = 0
, ее ты не должен воспринимать как аксиому.
тебя лишь просят отыскать равносильную ей формулу [math]A[/math]. доказательство равносильности [math]\vdash A \leftrightarrow f(x)=0[/math] будет решением задачи. а сама [math]A[/math] - это ответ на задачу. естественно, ответ на задачу требуется не абы какой, а в определенном удобном виде, например, на языке теории множеств.
обозначение [math]\vdash_T B[/math] означает доказательство формулы [math]B[/math] в теории [math]T[/math]. в нашем случае доказательство - это всего лишь цепочка твоих обоснованных рассуждений. если угодно, записанная словами естественного языка и математическими знаками.
Аноним 03/06/24 Пнд 21:51:58 115331 422
Снимок экрана 2[...].png 284Кб, 1270x434
1270x434
Ребят привет, я опять к вам, давайте обсудим задачу 118.


Вариант 1.
Пусть
P(x) = ax^p + ... + bx^q
Q(x) = cx^r + ... + dx^s
p > q > 0, r > s > 0.
При их перемножении точно останутся как минимум два одночлена: abx^p+1 и cdx^r+s (остальные могут сократиться или привестись как подобные слагаемые). Мы получили что нет, не может.

Вариант 2, менее строгий.
Пусть P(x) и Q(x) многочлены в стандартном виде с числом слагаемых больше 2. Они точно будут иметь ненулевой корень, в отличие от одночленов, которые имеют только один корень, равный нулю.
Очевидно, при их перемножении число корней складывается, так что они будут иметь как минимум два ненулевых корня, а значит они не могут быть одночленами.
Аноним 03/06/24 Пнд 23:59:25 115333 423
>>115331
>Вариант 2, менее строгий
Посмотри на $p(x)=x^3-x^2+x$ и $q(x)=x^2+x$. Сколько действительных корней у $p$, $q$, $pq$?
Идея первого варианта правильная, но можно написать аккуратнее.
Аноним 04/06/24 Втр 00:01:05 115334 424
>>115331
нет, не может.
у указанного произведения свободный член должен быть равен $0$, но тогда и у сомножителей свободные члены равны $0$, тогда переменную $x$ можно вынести и мы приходим к исходному условию, но для многочленов, у которых степень на $1$ ниже

продолжая, мы придём к тому, что исходные многочлены должны быть одночленами, иначе у произведения членов больше, чем один
Аноним 04/06/24 Втр 02:05:45 115335 425
>>115331
>>115334
Чому не может то? Вполне может, если брать например Z/4.
Аноним 04/06/24 Втр 04:10:14 115336 426
>>115335
Ты невнимательно прочёл задание. Очевидно, имеется в виду область целостности, иначе бы ответ к номеру 117 был бы другим. В следующий раз будь аккуратнее, когда влезаешь в беседу более умных людей
Аноним 04/06/24 Втр 04:25:37 115337 427
Чем при чтении дополнить Алюффи как первой обстоятельной книги по алгебре? Хочется чего-то более приземленного, вроде Herstein - Topics in Algebra (но там мало тем). Бурбаки и прочие Лэнги лесом.
Аноним 04/06/24 Втр 04:52:39 115338 428
>>115328
Большинство олимпиадников дропают математику.
>>115337
Edwards Galois Theory
Аноним 04/06/24 Втр 07:57:58 115339 429
>>115338
>Edwards Galois Theory
Нету практически никаких тем. Но полистаю.
Аноним 04/06/24 Втр 09:40:21 115340 430
IMG202406040938[...].jpg 3617Кб, 3000x4000
3000x4000
Тут можно было попроще решить?
Аноним 04/06/24 Втр 09:41:01 115341 431
И были ли неравно сильные переходы
Аноним 04/06/24 Втр 10:30:38 115343 432
>>115337
книга Алюффи безобразная, в особенности в качестве первой книги по алгебре. её можно использовать местами как справочник, если что-то конкретное не очень понятно, или как сборник примеров на универсальное свойство, но учиться по ней...

Лэнг значительно лучше, я считаю

в качестве первой книги я бы посоветовал Винберга
Аноним 04/06/24 Втр 12:59:07 115346 433
image.png 768Кб, 1529x464
1529x464
>>115340
Неправильный минус. Ну и переход к системе после этого необоснованный, т.к. найдя наборы из альфа и бэта, ей удовлетворяющие, получатся какие-то, но не обязательно все возможные решения изначального уравнения.
Ну и проще можно было, конечно - не делать замену, а заметить, что m = 2 явл. корнем и дальше поделить многочлен на (m-2)
Наша мудрыя советы Аноним 04/06/24 Втр 13:04:13 115347 434
привет!
советуем вам прочитать следующие книги.
они развивают общематематическую культуру и навыки доказательства

Гельфанд, Шень - Алгебра
Гельфанд - Функции и Графики
Гельфанд - Тригонометрия
Гельфанд - Метод координат
Успенский - Простейшие примеры математических доказательств
Шень - О математической строгости и школьном курсе математики
Serge Lang - Basic mathematics
Nicholas Smith - Logic. Laws of truth
Виленкин - Рассказы о множествах
Richard Hammack - Book of proof
Дэниэл Веллеман - Искусство доказательства в математике
Канель-Белов, Ковальджи - Как решают нестандартные задачи
Allendoerfer and Oakley - Principles of mathematics
Paul Zeitz - The Art and Craft of Problem Solving
Douglas Smith - A transition to advanced mathematics
Курант, Роббинс - Что такое математика?
Ian Stewart - Concepts of modern mathematics
Paul Halmos - Naive set theory.
Jay Cummings - Proofs
Курс видеолекций 3blue1brown по калькулюсу и линалу
Аноним 04/06/24 Втр 13:51:29 115349 435
Я если хочу получить больше интуиции и мотиваций, касающихся матана, то мне что, не остается ничего кроме совмещения дурацких книжек по Calculus с нормальными по анализу? А есть что-нибудь, где все вместе - и доказательства, и объяснения?
Аноним 04/06/24 Втр 15:07:44 115351 436
>>115346
ну я подбираю альфа и бета такие, что 3альфабета-12=0 (вместе с тем, что альфа + бета = m)
если указать это, переход равносильный? Если нет, то что ещё нужно указать
Аноним 04/06/24 Втр 17:04:01 115352 437
>>115351
>3альфабета-12=0 (вместе с тем, что альфа + бета = m)
это система уравнений, можно исследовать решения
Аноним 04/06/24 Втр 19:08:03 115353 438
>>115343
Я по Пинтеру учился сначала, мне нравился.
Ещё помню тут анон какую-то книгу рекомендовал. Помню она была на английском, но ориг с немецкого, вроде. И там очень много было про построение циркулем/линейкой. Жаль не помню автора.
Аноним 04/06/24 Втр 20:01:52 115354 439
>>115353
Нашёл каким-то чудом. Автор Lorenz. Кажется она.
Аноним 04/06/24 Втр 21:30:16 115355 440
Что почитать чтобы быстро научиться решать сложные задачи по матану? Где нужно не просто интеграл или производную взять а именно что-то проверить или доказать
Аноним 04/06/24 Втр 21:30:52 115356 441
>>115355
Или видос по теме какой нибудь скиньте я хз, даже лучше будет
Аноним 04/06/24 Втр 22:49:10 115357 442
>>115355
Решения задач на доказательства по матану, например в Виноградовой, ещё книжку примеры и контрпример в
Аноним 04/06/24 Втр 22:51:35 115358 443
>>115357
Контрпримеры в анализе гелбаума, чёт заклинило
Аноним 04/06/24 Втр 23:33:11 115359 444
>>115358
>>115357
Мне нравятся по содержанию талмуды Пойа но они длинные. Есть что почитать чтоб за день управиться и понять?
Аноним 04/06/24 Втр 23:41:32 115360 445
>>115262
Какая у детерминанта функториальность? Про то, что его можно рассматривать как естественное преобразование я слышал, про это — нет.
Аноним 04/06/24 Втр 23:46:26 115361 446
>>115359
Ну от них ты не научишься решать сложные задачи по матану, они скорее нужны чтобы найти эффективный способ изучения математики. А если отвечать на твой вопрос, то нет никакой коротенькой методички прочитав которую ты сложные задачи научишься решать. Тут только брать и решать
Аноним 05/06/24 Срд 05:01:06 115362 447
Реально ли за 10 месяцев подготовиться к экзамену в ШАД, имея из математического образования лишь диплом СПО и почти нулевые реальные знания (ну может могу производную найти и СЛАУ решить методом Гаусса)? Или о таком даже спрашивать смешно?
https://yastatic.net/s3/academy/shad/enroll/books_list.pdf
Аноним 05/06/24 Срд 05:22:34 115363 448
ASTOUNDING 1 2 [...].png 519Кб, 853x629
853x629
ASTOUNDING 1 2 [...].png 480Кб, 853x628
853x628
Not -112.png 330Кб, 853x627
853x627
triple-parenthe[...].png 117Кб, 782x670
782x670
Вы выиграли в лотерею одно яблоко плюс два яблока плюс три яблока и так до бесконечности. Вопрос. Сколько у вас будет яблок в итоге?

Готовы? Воздуха набрали? Ответ. У вас будет 0 яблок и вы ещё останетесь должны 1/12 яблока АХАХАХАХА

Если мне скажут что я выиграл в такую лотерею, то я эти яблоки посчитаю вот так
1+2+3+... = 1+(1+1)+(1+1+1)+... = 1+1+1+1+1+1+...
и никому ничего не буду должен (ебало того кто придумал эту лотерею на 4 пике)

Очень интересно что же он мне ответит?
Аноним 05/06/24 Срд 05:30:59 115364 449
Аноним 05/06/24 Срд 06:55:43 115365 450
Аноним 05/06/24 Срд 07:57:11 115366 451
>>115365
Тогда полагаю, что нереально. Я с нуля 2 года готовился к поступлению на матмех, один из них прямо въебывал.
Аноним 05/06/24 Срд 08:23:54 115367 452
>>115363
пошел вон отсюда подонок юдофоб
Аноним 05/06/24 Срд 08:29:42 115368 453
>>115347
Всё жду умельца, который составит огромный такой талмуд последовательно доказанных теорем какого-нибудь раздела.
Аноним 05/06/24 Срд 09:38:10 115370 454
>>115368
Был такой, его звали Бурбаки.
Аноним 05/06/24 Срд 10:03:36 115371 455
>>115370
а стоит ли изучать бурбаки в 2024?
Аноним 05/06/24 Срд 10:20:08 115372 456
>>115363
следущая лотерея: ты выйграл 1000 мнимых долларов, но не переживай что они мнимые, через месяц ты получишь их в квадрате
Аноним 05/06/24 Срд 10:23:43 115373 457
image.png 1503Кб, 1280x720
1280x720
Аноним 05/06/24 Срд 10:33:26 115374 458
>>115368
и кстати что скажешь о моей ебейшей подборочке?
если по всем этим книжкАм пройтись досконально, это апнет мою mathematisches kultur?
Аноним 05/06/24 Срд 10:43:46 115375 459
>>115352
так разве мы не это сделали?
Аноним 05/06/24 Срд 11:42:41 115376 460
>>115371
У них нормальный том по алгебре.
Аноним 05/06/24 Срд 12:39:09 115377 461
>>115363
Да расслабься, чел. Это говно само будет обоссано и забыто.
Средний iq по планете падает, всякие задроты не плодятся и вымирают. Зато обладатели "нормальных" генов плодятся мощно.
Лет через 50 повсюду будут халифаты, где эта маняматика нахуй не сдалась. А большую часть того, что написано по ней, просто никто уже не поймёт.
И не надейся, что элиты будут "умными". Глянь на нынешних. Деградация коснётся всех.
Я ещё жду, когда идиоты догадаются, что вместо роботов выгоднее рабов использовать
Аноним 05/06/24 Срд 15:32:54 115379 462
>>115377
Советую тебе выпилиться, тогда "iq по планете" раза в два вырастет.
Аноним 05/06/24 Срд 15:45:04 115380 463
>>115379
Хуя у педика жопу подпалило, лол
Аноним 05/06/24 Срд 17:43:49 115381 464
>>115380
Не проецируй, умный ты наш.
Аноним 05/06/24 Срд 17:59:02 115382 465
Через какое время нужно дропать задачу, если ее не получается решить? Если ты самоучка.
Аноним 05/06/24 Срд 18:51:45 115383 466
Барс.jpg 224Кб, 761x1035
761x1035
Барс2.jpg 339Кб, 687x1080
687x1080
Анончики, помогите пожалуйста. Можно ли взять школьный учебник Барсукова (6-7) 1959г и прилагающиеся сборник задач к нему же. Вот ссылочка, посмотрите пожалуйста, актуально ли к изучению по нему?
https://vk.com/stalins_bukvar?w=wall-142811591_40783
Аноним 05/06/24 Срд 19:17:30 115384 467
>>115383
Вполне можно. Никаких принципиальных отличий между совковыми учебниками и несовковыми нет, это все миф шизов старых, какой учебник по школьной программе нравится такой и бери
Аноним 05/06/24 Срд 19:28:06 115386 468
>>115382
Нужно не дропать, а решение смотреть. Иначе есть шанс вообще ничему не научиться.
Аноним 05/06/24 Срд 20:49:39 115388 469
photo2024-06-05[...].jpeg 15Кб, 760x94
760x94
Товарищи, я опять нарешал!
Кажется, что решение не очень строгое. Некоторые моменты я понял по наитию, а как их строго объяснить - моя не знать, такие я выделил спойлером..

Пусть P(x) - многочлен степени n. Распишем разность всех одончленов, входящих в многочлен (свободный член сократится), каждый при соответствующей степени.
По условию P(a)-P(b)=1 <=> qa^n - qb^n + la^(n-1) - lb^(n-1) + ... + ma - mb = 1, где q,l,m - целые <=> q(a - b)Q(x) + l(a - b)L(x) + ... + m(a-b)M(x) = 1 Q(x), L(x), M(x) - многочлены с целочисленными коэффициентами степени n-1. Я много решал задачек, и научился алгоритму, по которому из любого многочлена вида a^n - b^n (и даже a^n + b^n, но в случае n = 2 надо подключать комплексные числа) можно вынести a-b. Оно понятно, ведь при a-b=0, значит a-b его множитель. Но как это строго обосновать?. Вынесем (a-b): (a-b)[qQ(x) + lL(x) + ... + mM(x)] = 1.
По условию многочлен с целыми коэффициентами, поэтому [q
Q(x) + lL(x) + ... + mM(x)] - какая-то линейная комбинация, точно являющаяся каким-то целым числом. Оно при умножении на (a-b) даёт единицу. Очевидно, что два целых числа могут дать при умножении единицу тогда и только тогда, когда они оба равны ±1. Получается, |a − b| = 1, чтд.
Аноним 05/06/24 Срд 21:01:19 115389 470
>>115388
Тут из того что a^n-b^n делится на a-b следует что P(a)-P(b) делится на a-b, вот и получаешь что abs(a-b)=1 так как a-b делитель 1
Аноним 05/06/24 Срд 21:05:40 115390 471
>>115389
Аааа! Вот я дурачина, всё гораздо проще. Спасибо анон!
Аноним 05/06/24 Срд 22:38:08 115392 472
>>115370
Слишком больны неоправданным формализмом. Дальше "Теории множеств" не стал читать.
Аноним 05/06/24 Срд 22:49:21 115393 473
>>115374
>Гельфанд - Тригонометрия
Надо что-то лучше.
Методика доказывания тригонометрических функций суммы (разности) углов у них крайне выматывающая, а ведь могли по простому векторами всё объяснить.
Но обязательно найдутся мелкобуквенные умники, которые начнут пояснять за удобство мнимых чисел.
Аноним 06/06/24 Чтв 00:40:25 115394 474
>>29047 (OP)
А есть ли книги чисто прикладного, житейского характера по математике?
Аноним 06/06/24 Чтв 05:13:10 115395 475
>>115393
Ты книгу открывал? Там формулы суммы выводят из скалярного произведения векторов, комплексные числа там не используют. Хотя надо.
Аноним 06/06/24 Чтв 06:29:09 115396 476
>>115394
Легкая математика покера, Роя Раундера
Аноним 06/06/24 Чтв 10:13:18 115399 477
>>115367
Ну это понятно что он мне так ответит.
Хотелось бы услышать от него аргументы почему я не прав.

>>115372
Спасибо, подожду не месяц, а три (ну или 3+4n месяцев)
либо поверну мнимые даллары на 180 градусов в мнимом пространстве
либо задоначу их в фонд поддержки демократии
А вообще на такую лотерею мало кто поведётся. А вот бесконечно увеличивающеюся количество яблок выглядит очень привлекательно, так что в этой теме нужно разобраться.

>>115377
Не совсем понял кем ты меня считаешь. Я хоть и далёк от математики, но математику уважаю, и вполне допускаю что там может быть -1/12 (а может и не быть), просто в любых темах всегда стараюсь вызвать спор, так как в споре рождается истина.
А в целом с твоим постом согласен. У группы влиятельных богатых людей есть влажные мечты о том чтобы в будущем все были одинаковыми и послушными, сидели по домам, ели жуков, трахали роботов, и были рады всему говну которое пихают в их глаза и уши. А пока мы к этому идём, нужно навязать всем единую культуру, смешать всех в одну кучу, дегенератов нужно сделать равными элите, а чтобы они не выбивались на фоне остальных, нужно весь мир опустить на их уровень. Всех несогласных нужно отсеять. Халифаты в европе нужны для закрепления там муслимов, потом они станут гей-халифатами привлекательными для пидорасов всех рас. Также нужно создать выдуманную глобальную угрозу(например увеличение населения ведущее к глобальному потеплению), затем нужно создать единое правительство которое будет "спасать" планету от этой угрозы. А чтобы все поверили в эту глобальную угрозу нужно создать искуственный дефицит и разогнать инфляцию, например платить безработным мигрантам, и тем кто весь день пишет "правильные" посты в твитторе и на реддите. Больше всех нужно платить тем кто протестует против фермерства и добычи нефти.
Ой, что-то я разошёлся. Это же всё ультраправые антисемитские теории заговора. Зачем я трачу на них своё время? Аж голова заболела. Пойду лучше и дальше рубиться в дотку, слушать гачи-треки и смотреть куколд-порнуху, пока мои родители трудятся на работе чтобы у детей мигрантов было светлое будущее. Я так рад что не продолжу свой род ведь я спасаю планету от глобального потепления. И мне плевать что жуков можно лучше скормить курочке у которой для этого лучше предназначен желудок. Если все будут есть жуков, то я чё, не такой как все чтоли?
Аноним 06/06/24 Чтв 10:16:50 115400 478
Аноним 06/06/24 Чтв 15:18:26 115401 479
Начал учить математику, но мозг вообще не работает, сижу над задачей по полтора-два часа и не могу понять, как её решить. Легко даются только так задачи, в которых нужно по алгоритму всё сделать. Как это лечить?
Аноним 06/06/24 Чтв 15:29:30 115402 480
>>115401
(предполагая, что проблема в процессе обучения, а не в плохом распорядке дня, рационе, сне, и проч.)
Обсуждай задачу. Задавай вопросы. Или со сверстниками, или с самим собой. Гугли свои вопросы на нормальных площадках вроде стэкиксчендж.
Когда читаешь учебник, всё воспринимай как задачу. Определение - задача (внезапно). Лемма, теорема, небрежный комментарий автора - всё задачи тебе подумать.
Аноним 06/06/24 Чтв 15:56:32 115403 481
>>115402
Я решаю листки, там только задачи без комментариев, лемм и теорем. А на вопросы у меня нет времени, я стараюсь интенсивно учиться, а не по пару часов в день. И если мне ответят на вопрос - это же подсказка, так нечестно.
Аноним 06/06/24 Чтв 16:16:57 115404 482
>>115403
Так суть не в том чтобы якобы "честно" решить все задачи, а в том чтобы научиться их решать. При обучении по листочкам важную роль играют преподы и другие ученики у которых собственно и можно получить подсказку или само решение.
Аноним 06/06/24 Чтв 16:42:28 115405 483
>>115395
>Там формулы суммы выводят из скалярного произведения векторов, комплексные числа там не используют.
Ты сообщение, на которое отвечаешь, читал?
>Методика доказывания тригонометрических функций суммы (разности) углов у них крайне выматывающая, а ведь могли по простому векторами всё объяснить.
Аноним 06/06/24 Чтв 16:57:53 115406 484
>>115403
Тебе не обязательно должны отвечать на вопрос. Суть в том, чтобы научиться рассуждать.
>интенсивно
Интенсивно не значит эффективно.
Аноним 06/06/24 Чтв 17:06:14 115407 485
Тут на доске, если не ошибаюсь, полгода назад начинали решать Давидовича-Пушкаря, как успехи?
Аноним 06/06/24 Чтв 17:18:03 115408 486
>>115407
Сначала в том чате четко соблюдали правила и общались только по делу. Потом админ-нацик проебал чат и новый админ-битард начал флудить в нём на пару с каким-то пиздюком. Я после этого вышел и забил хуй.
Аноним 06/06/24 Чтв 17:26:34 115409 487
>>115399
Почему в мае выпадал снег в Москве? Почему на средних полосах повысилась температура зимой аж до плюсовой?
Аноним 06/06/24 Чтв 17:29:19 115410 488
>>115401
Ты бы еще спросил как повысить свой интеллект. Никак.

У каждого индивида свой потолок комплексности, выше которого он никогда не продвинется. У кого-то инсайт прыгает на пару ходов, а у кого-то на целую тысячу. У кого-то он случается каждые пять минут, а кому-то приходится ждать неделю.

В заучивание чужой математики способен 1% белого населения, а в создание своей - 0,01%. Это никак не вылечить.
Аноним 06/06/24 Чтв 17:32:06 115411 489
>>115410
>белого населения
Это не ты создал группу, о которой писали выше?
Аноним 06/06/24 Чтв 18:03:30 115412 490
>>115401
Давай пример задачи
Аноним 06/06/24 Чтв 18:45:41 115413 491
Аноним 06/06/24 Чтв 19:02:36 115414 492
image.png 34Кб, 790x265
790x265
>>29047 (OP)
Помогите найти материалы на русском по этой теме,сам нихуя не нашел.Вообще не уверен если это учится в школе
Примерный перевод задания: В декартовой системе координат заданы отрезки p и q в параметрической форме.
Аноним 06/06/24 Чтв 19:38:25 115415 493
>>115414
Ангем называется
Аноним 06/06/24 Чтв 21:21:29 115416 494
Аноним 07/06/24 Птн 00:08:22 115418 495
Товарищи, занимающиеся математикой, а вам сложно общаться и сходиться с другими людьми? Вас считают странноватыми чудиками?
Аноним 07/06/24 Птн 01:36:44 115419 496
>>115418
По моему опыту, есть два типа хороших математиков:
1) умные более-менее нормисы без серьезных психических/поведенческих/социальных отклонений;
2) гениальные аутисты не от мира сего;
3) гениальные нормисы.
Первый тип встречается намного чаще и составляет основную массу работающих математиков, второй тип, наверное, самый редкий. Те, кто по середине - странноватые, социально неловкие люди, умные, но не гениальные, - как раз в долгосрочной перспективе преуспевают меньше всего, если не становятся представителями первого типа.
Аноним 07/06/24 Птн 04:00:56 115420 497
https://mathter.pro/pesochnica/2_7_uravneniya_i_neravenstva_s_modulem.html
Так, для того чтобы решить неравенство
$-4\lt -2x\leq1$
нужно все его части умножить на -1/2 и поскольку это число отрицательное, то «значки» неравенств следует «развернуть» в противоположном направлении, после чего переписать результат «справа налево»:
$-{\frac{1}{2}}\leq x\lt 2$
В чём решение заключается? Автор учебника же просто переписал неравенство в другую форму.
Аноним 07/06/24 Птн 04:02:50 115421 498
>>115420
Или тут как в задачках на поиск подобных слагаемых, нужно максимально упростить, в данном случае так, чтобы x был без коэффициента?
Аноним 07/06/24 Птн 05:42:05 115422 499
>>115419
Я не могу вспомнить ни одного западного математика аутиста. Думаю мяукающие долбаебы типа Дурова, или аутяры Перельманы, это чисто продукт советской матшколы с ёблей в жопу. На западе все гениальные это ультранормисы и не редко многодетные. У Гротендика вообще куча женщин было, детей, и быдлу на улице в ёбыч мог прописать.
Аноним 07/06/24 Птн 06:02:43 115423 500
>>115420
Прочитай определение решения неравенства
Аноним 07/06/24 Птн 06:24:38 115424 501
>>115420
Так у тебя учебник для 2 класса. Чего ты хотел. Спроси ещё в чем заключается решение уравнения
-2х = 4
х = -2
тут тоже уравнение просто в другую форму переписали или всеже нашли значение х?
Ответить в тред Ответить в тред

Check this out!

Настройки X
Ответить в тред X
15000
Добавить файл/ctrl-v
Стикеры X
Избранное / Топ тредов